SlideShare a Scribd company logo
1 of 67
Viết thuê đề tài giá rẻ trọn gói - KB Zalo/Tele : 0973.287.149
Luanvanmaster.com – Cần Kham Thảo - Kết bạn Zalo/Tele : 0973.287.149
ĐẠI HỌC THÁI NGUYÊN
TRƯỜNG ĐẠI HỌC KHOA HỌC
PHẠM THANH LINH
MỘT SỐ LỚP PHƯƠNG TRÌNH HÀM
TRONG SỐ HỌC
LUẬN VĂN THẠC SĨ TOÁN HỌC
Thái Nguyên -
Viết thuê đề tài giá rẻ trọn gói - KB Zalo/Tele : 0973.287.149
Luanvanmaster.com – Cần Kham Thảo - Kết bạn Zalo/Tele : 0973.287.149
Mục lục
Lời cảm ơn iii
Mở đầu 1
1 Lớp hàm số học cơ bản 3
1.1 Hàm số học . . . . . . . . . . . . . . . . . . . . . . . . . . . . . . 3
1.1.1 Hàm nhân tính . . . . . . . . . . . . . . . . . . . . . . . . 3
1.1.2 Hàm nhân tính mạnh . . . . . . . . . . . . . . . . . . . . . 11
1.2 Hàm số xác định trên tập các số nguyên . . . . . . . . . . . . . . . 11
1.2.1 Hàm cộng tính trên tập các số nguyên . . . . . . . . . . . . 11
1.2.2 Hàm nhân tính trên tập các số nguyên . . . . . . . . . . . . 11
1.2.3 Lớp hàm tuần hoàn, phản tuần hoàn cộng tính, nhân tính . . 12
1.3 Một số bài tập áp dụng . . . . . . . . . . . . . . . . . . . . . . . . 14
2 Các phương trình hàm số học 20
2.1 Hàm chuyển đổi các phép tính số học . . . . . . . . . . . . . . . . 20
2.1.1 Hàm chuyển đổi phép cộng thành phép cộng . . . . . . . . 20
2.1.2 Hàm chuyển đổi phép cộng thành phép nhân . . . . . . . . 21
2.1.3 Hàm chuyển đổi phép nhân thành phép cộng . . . . . . . . 22
2.2 Các dạng toán xác định dãy số liên quan . . . . . . . . . . . . . . . 22
2.3 Các bài tập áp dụng . . . . . . . . . . . . . . . . . . . . . . . . . . 27
3 Các dạng toán liên quan 33
3.1 Phương trình hàm trên N và trên Z . . . . . . . . . . . . . . . . . . 33
Viết thuê đề tài giá rẻ trọn gói - KB Zalo/Tele : 0973.287.149
Luanvanmaster.com – Cần Kham Thảo - Kết bạn Zalo/Tele : 0973.287.149
ii
3.1.1 Lớp các bài toán áp dụng nguyên lý quy nạp toán học . . . . 33
3.1.2 Lớp các bài toán áp dụng nguyên lí cực hạn . . . . . . . . . 42
3.1.3 Lớp các bài toán áp dụng hệ đếm cơ số . . . . . . . . . . . 46
3.1.4 Lớp các bài toán áp dụng các tính chất số học . . . . . . . . 53
3.1.5 Lớp các bài toán áp dụng các tính chất dãy số . . . . . . . . 62
3.1.6 Lớp các bài toán áp dụng các tính chất hàm số . . . . . . . . 66
3.2 Phương trình hàm trên Q . . . . . . . . . . . . . . . . . . . . . . . 73
Kết luận và Đề nghị 81
Tài liệu tham khảo 82
Viết thuê đề tài giá rẻ trọn gói - KB Zalo/Tele : 0973.287.149
Luanvanmaster.com – Cần Kham Thảo - Kết bạn Zalo/Tele : 0973.287.149
iii
Lời cảm ơn
Luận văn này được thực hiện và hoàn thành tại Trường Đại học Khoa
học - Đại học Thái Nguyên.
Đầu tiên em xin bày tỏ lòng biết ơn sâu sắc nhất tới người thầy đáng kính
GS.TSKH. Nguyễn Văn Mậu - Trường Đại học Khoa học Tự nhiên - Đại học
Quốc gia Hà Nội. Thầy đã dành nhiều thời gian hướng dẫn và giải đáp các thắc
mắc trong suốt quá trình xây dựng đề cương, làm và hoàn thiện luận văn.
Em xin gửi lời cảm ơn chân thành nhất đến các Thầy cô khoa Toán, phòng
Đào tạo sau Đại học, trường Đại học Khoa học - Đại học Thái Nguyên, cùng các
Thầy cô giáo tham gia trực tiếp giảng dạy lớp cao học khóa 1/2014 - 1/2016.
Đồng thời tôi xin gửi lời cảm ơn tới tập thể lớp K7C Cao học Toán - Đại học
Khoa học đã động viên giúp đỡ tôi trong quá trình học tập và làm luận văn này.
Em xin chân thành cảm ơn!
Thái Nguyên, 2015 Phạm Thanh Linh
Học viên Cao học Toán K7C,
Trường ĐH Khoa học - ĐH Thái Nguyên
Viết thuê đề tài giá rẻ trọn gói - KB Zalo/Tele : 0973.287.149
Luanvanmaster.com – Cần Kham Thảo - Kết bạn Zalo/Tele : 0973.287.149
1
Mở đầu
Phương trình hàm là một lĩnh vực quan trọng của giải tích. Bài toán giải
phương trình hàm có lẽ là một trong những bài toán lâu đời nhất của giải tích.
Nhu cầu giải phương trình hàm xuất hiện ngay khi bắt đầu có lí thuyết hàm
số. Nhiều phương trình hàm xuất phát từ nhu cầu thực tế của Toán học hoặc
của các ngành khoa học khác. Các nhà toán học đã có công nghiên cứu và
đặt nền móng cho phương trình hàm phải kể đến: Nicole Oresme, Gregory of
Saint-Vincent, Augusstin-Louis Cauchy, Carl Friedrich Gauss, D’Alembert . . .
Ngày nay ở nước ta phương trình hàm được giảng dạy theo chuyên đề ở các
trường THPT chuyên. Các dạng toán phương trình hàm trong số học là dạng toán khó
thường xuất hiện trong các kì thi học sinh giỏi cấp tỉnh, thành phố, cấp quốc gia, khu vực
và quốc tế. Một phương trình hàm bao gồm ba thành phần chính: Tập nguồn, tập đích,
phương trình hay hệ phương trình hàm. Từ ba thành phần này có sự phân loại tương
ứng. Phương trình hàm trên N, phương trình hàm trên Z, phương trình hàm trên Q,
phương trình hàm trên R · · · ; phương trình hàm với một biến tự do, hai biến tự do,
nhiều biến tự do, phương trình hàm chuyển đổi các giá trị trung bình · · · ; phương trình
hàm trên lớp hàm khả vi, phương trình hàm trên lớp hàm liên tục · · ·
Việc xác định rõ cấu trúc và tính chất của tập nguồn, tập đích và các điều kiện ràng
buộc sẽ quyết định sự thành công hay thất bại khi giải phương trình hàm. Điều này
có thể thấy rõ qua phương trình hàm Cauchy. Bài toán tổng quát tìm tất cả các hàm
số f : R → R thỏa mãn phương trình f (x + y) = f (x) + f (y) với mọi x, y ∈ R theo một
nghĩa nào đó không có lời giải, thế nhưng với giới hạn tập nguồn, tập đích, các tính
Viết thuê đề tài giá rẻ trọn gói - KB Zalo/Tele : 0973.287.149
Luanvanmaster.com – Cần Kham Thảo - Kết bạn Zalo/Tele : 0973.287.149
chất của hàm số (đơn điệu, liên tục · · · ) thì phương trình hàm này giải được trọn
vẹn. Trong luận văn này tôi chỉ xin trình bày về một lớp phương trình hàm mà
Viết thuê đề tài giá rẻ trọn gói - KB Zalo/Tele : 0973.287.149
Luanvanmaster.com – Cần Kham Thảo - Kết bạn Zalo/Tele : 0973.287.149
2
tập nguồn xác định trên N, Z, Q. Trên thực tế khi tìm hiểu lớp các phương trình
hàm này xuất hiện rất nhiều trong các kì thi Olympic toán các nước, khu vực và
quốc tế. Xuất phát từ thực tế đó, dưới sự định hướng và hướng dẫn nhiệt tình
của GS.TSKH. Nguyễn Văn Mậu, tôi đã tiến hành nghiên cứu về đề tài "Một số
lớp phương trình hàm trong số học" nhằm mục đích học tập và nghiên cứu sâu
về một chuyên đề khó của toán sơ cấp. Cấu trúc luận văn gồm 3 chương.
Chương 1. Lớp hàm số học cơ bản
Trong chương này sẽ trình bày định nghĩa, các tính chất của các hàm số
học cơ bản và một số ứng dụng của chúng vào việc giải các bài toán sơ cấp.
Chương 2. Các phương trình hàm số học
Trong chương này sẽ trình bày về hàm chuyển đổi các phép tính số
học, các dạng toán xác định dãy số liên quan và các bài tập áp dụng.
Chương 3. Các dạng toán liên quan
Trong chương này sẽ trình bày các dạng toán từ các đề thi Olympic các
nước và quốc tế liên quan đến tính toán, ước lượng và các tính chất số
học (nguyên tố, chính phương, tính đơn điệu, tính tuần hoàn. . . ) của các
hàm số trên các tập số tự nhiên, tập số nguyên và tập số hữu tỷ.
Dù đã nghiêm túc nghiên cứu và rất cố gắng thực hiện luận văn, nhưng do
năng lực của bản thân cùng nhiều lý do khác, luận văn chắc chắn không tránh
khỏi những thiếu sót. Kính mong được sự góp ý của các Thầy cô, các bạn và
các anh chị đồng nghiệp để luận văn này hoàn chỉnh và có nhiều ý nghĩa hơn.
Thái Nguyên, ngày 28 tháng 11 năm 2015
Phạm Thanh Linh
Học viên Cao học Toán lớp C, khóa 01/2014 - 01/2016
Chuyên ngành phương pháp Toán sơ cấp
Trường Đại học Khoa học - Đại học Thái Nguyên
Viết thuê đề tài giá rẻ trọn gói - KB Zalo/Tele : 0973.287.149
Luanvanmaster.com – Cần Kham Thảo - Kết bạn Zalo/Tele : 0973.287.149
Email: bobaki2010@gmail.com
Viết thuê đề tài giá rẻ trọn gói - KB Zalo/Tele : 0973.287.149
Luanvanmaster.com – Cần Kham Thảo - Kết bạn Zalo/Tele : 0973.287.149
3
Chương 1
Lớp hàm số học cơ bản
Trong chương trình phổ thông, các bài toán số học đóng vai trò quan
trọng trong việc hình thành tư duy toán học. Việc tìm hiểu và sử dụng các
hàm số học đã giải quyết được những lớp bài toán cơ bản trong các bài
toán sơ cấp và lớp các bài toán phương trình hàm trong số học.
Trong chương này sẽ trình bày định nghĩa, các tính chất của các hàm số
học cơ bản và một số ứng dụng của chúng vào việc giải các bài toán sơ cấp.
1.1 Hàm số học
Định nghĩa 1.1 ([9]). Hàm số học tức là hàm xác định trên tập các số nguyên dương.
1.1.1 Hàm nhân tính
Định nghĩa 1.2 ([9]). Một hàm số học f được gọi là nhân tính nếu với mọi
m, n nguyên tố cùng nhau, ta có đẳng thức
f (mn) = f (m)f (n).
Những ví dụ đơn giản nhất về hàm nhân tính là f (n) = n và f (n) = 1.
Ngoài các hàm nhân tính đơn giản kể trên thì ta phải kể đến các hàm nhân
tính quan trọng khác như : Phi-hàm Euler, hàm sinh bởi các ước, hàm tổng
Viết thuê đề tài giá rẻ trọn gói - KB Zalo/Tele : 0973.287.149
Luanvanmaster.com – Cần Kham Thảo - Kết bạn Zalo/Tele : 0973.287.149
các ước số, hàm số các ước số, hàm Mobius. Trong các hàm số học, hàm
Euler mà ta định nghĩa sau đây có vai trò rất quan trọng.
Viết thuê đề tài giá rẻ trọn gói - KB Zalo/Tele : 0973.287.149
Luanvanmaster.com – Cần Kham Thảo - Kết bạn Zalo/Tele : 0973.287.149
4
Định nghĩa 1.3 ([9]). Giả sử n là số nguyên dương. Phi-hàm Euler ϕ(n) là hàm số
học có giá trị tại n bằng số các số không vượt quá n và nguyên tố cùng nhau với n.
Ví dụ 1.1. Từ định nghĩa ta có
ϕ(1) = 1, ϕ(2) = 1, ϕ(3) = 2, ϕ(4) = 2, ϕ(5) = 4,
ϕ(6) = 2, ϕ(7) = 6, ϕ(8) = 4, ϕ(9) = 6, ϕ(10) = 4.
Từ định nghĩa trên đây, ta có ngay hệ quả trực tiếp.
Hệ quả 1.1 ([9]). Số p là nguyên tố khi và chỉ khi ϕ(p) = p − 1.
Chứng minh. Nếu p là số nguyên tố thì với mọi số nguyên dương nhỏ hơn p
đều nguyên tố cùng nhau với p. Do có p − 1 số nguyên dương như vậy nên ϕ(p) = p
− 1. Ngược lại, nếu p là hợp số thì p có các ước d, 1 < d < p. Tất nhiên p và d không
nguyên tố cùng nhau. Như vậy trong các số 1, 2, · · · , p − 1 phải có số không
nguyên tố cùng nhau với p, nên ϕ(p) < p − 2. Điều này trái với giả thiết ϕ(p) = p − 1.
Định nghĩa 1.4 ([2]). Một tập hợp A nào đó được gọi là một hệ thặng dư đầy
đủ (mod n) chính là n số mà không có hai số nào đồng dư nhau theo (mod n).
Ví dụ 1.2. Ta có A = {0, 1, 2, · · · , n − 1} là một hệ thặng dư đầy đủ theo
(mod n). Hay đơn giản hơn tập B = {0, 1, 2, 3, 4, 5, 6, 7} và C = {2, 3, 4, 5,
6, −7, 8} là các hệ thặng dư đầy đủ theo (mod 8).
Định nghĩa 1.5 ([2]). Một hệ thặng dư thu gọn môđulô n là tập hợp gồm
ϕ(n) số nguyên sao cho mỗi phần tử của tập hợp đều nguyên tố cùng nhau
với n và không có hai phần tử khác nhau nào đồng dư môđulô n.
Ví dụ 1.3. Tập hợp {1, 3, 5, 7} và tập {3, −7, 5, 7} là các hệ thặng dư thu
gọn (mod 8).
Định lí 1.1 ([2]). Giả sử r1, r2, . . . , rϕ(n) là một hệ thặng dư thu gọn (mod n),
Viết thuê đề tài giá rẻ trọn gói - KB Zalo/Tele : 0973.287.149
Luanvanmaster.com – Cần Kham Thảo - Kết bạn Zalo/Tele : 0973.287.149
a là số nguyên dương và (a, n) = 1. Khi đó tập ar1, ar2, · · · , arϕ(n) cũng là hệ
thặng dư thu gọn (mod n).
Viết thuê đề tài giá rẻ trọn gói - KB Zalo/Tele : 0973.287.149
Luanvanmaster.com – Cần Kham Thảo - Kết bạn Zalo/Tele : 0973.287.149
5
Chứng minh. Vì r1, r2, . . . , rϕ(n) là một hệ thặng dư thu gọn (mod n) nên
tập
1 2 · · · ϕ(n)
có ϕ(n) số. Ta chỉ cần chứng minh khi i = j thì ar
1 ∼ j
ar, ar , , ar = ar
.
.
(mod n). Thật vậy: Giả sử ari ≡ arj (mod n) suy ra a (ri − rj ) . n, vì (a, r) = 1
.
.
nên ri − rj . n suy ra ri ≡ rj (mod n), trái với r1, r2, . . . , rϕ(n) là hệ thặng dư
thu gọn ( Vì hệ thặng dư thu gọn không có hai số nào đồng dư nhau).
Ví dụ 1.4. Tập {1, 3, 5, 7 } là một hệ thặng dư thu gọn (mod 8). Do (3, 8) =
1 nên tập {3, 9, 15, 21} cũng là một hệ thặng dư thu gọn (mod 8) .
Định lí 1.2 (Định lí Euler).
(a, n) = 1. Khi đó aϕ(n)
≡ 1
Giả sử n là số nguyên dương và a là số nguyên
với (mod n).
Chứng minh. Xét hệ thặng dư thu gọn (mod n)
r1, r2, · · · , rϕ(n)
Suy ra ar1, ar2, · · · , arϕ(n) cũng là một hệ thặng dư thu gọn (mod n). Do đó ta có
ar1.ar2 . . . arϕ(n) ≡ r1.r2 . . . rϕ(n)(mod n)
⇒ a
ϕ(n)
.r1.r2 . . . rϕ(n) ≡ r1.r2 . . . rϕ(n) (mod n)
ϕ(n) .
− 1 r1.r2
.
⇒ a . . . rϕ(n) . n
ϕ(n) ..
⇒ a − 1 . n
⇒ a
ϕ(n)
≡ 1(mod n).
Chú ý 1.1. Ta có thể tìm nghịch đảo của (mod n) bằng cách sử dụng định
lí Euler. Giả sử (a, m) = 1 khi đó a.aϕ(n)−1
= aϕ(n)
≡ 1(mod n) trong đó
a
ϕ(n)−1
là nghịch đảo của a(mod n).
Ví dụ 1.5. 2
ϕ(9)−1
= 2
6−1
= 2
5
= 32 ≡ 5(mod 9) là ngịch đảo của 2(mod 9).
Hệ quả 1.2 ([2]). Nếu (a, b) = 1 thì aϕ(b)
+ bϕ(a)
≡ 1(mod ab).
Hệ quả 1.3 ([2]). Với (a, b) = 1 và n, v là hai số nguyên dương nào đó thì
Viết thuê đề tài giá rẻ trọn gói - KB Zalo/Tele : 0973.287.149
Luanvanmaster.com – Cần Kham Thảo - Kết bạn Zalo/Tele : 0973.287.149
a
nϕ(b)
+ b
vϕ(a)
≡ 1 (modab) .
Viết thuê đề tài giá rẻ trọn gói - KB Zalo/Tele : 0973.287.149
Luanvanmaster.com – Cần Kham Thảo - Kết bạn Zalo/Tele : 0973.287.149
6
Hệ quả 1.4 ([2]). Giả sử có k (k ≥ 2) số nguyên dương m1, m2, · · · , mk và
chúng nguyên tố với nhau từng đôi một. Đặt M = m1m2 . . . mk = mi.ti với i
= 1, 2, . . . , k ta có
t
n
1 + t
n
2 + · · · + t
n
k ≡ (t1 + t2 + · · · + tk)
n
(mod
M), với n nguyên dương.
Định lí 1.3 ([2]). Phi-hàm Euler là hàm nhân tính.
Chứng minh. Ta viết các số nguyên dương không vượt quá mn thành bảng sau:
1 m + 1 2m + 1 . . . (n − 1) m + 1
2 m + 2 2m + 2 . . . (n − 1) m + 2
3 m + 3 2m + 3 . . . (n − 1) m + 3
... ... ...... . . .
m 2m 3m . . . mn
Bây giờ giả sử r là một số nguyên không vượt quá m. Giả sử (m, r) = d > 1. Khi đó,
không có số nào trong dòng thứ r nguyên tố cùng nhau với mn, vì mỗi phần tử của
dòng đó đều có dạng km + r, trong đó 1 ≤ k ≤ n − 1, d| (km + r), vì d|m, d|r. Vậy để
các số trong bảng mà nguyên tố cùng nhau với mn, ta chỉ cần xét các dòng thứ
r với (m, r) = 1. Ta xét một dòng như vậy, nó chứa các số r, m+r, . . . , (n − 1) m+r.
Vì (m, r) = 1 nên mỗi số nguyên trong dòng đều nguyên tố cùng nhau với n. Như vậy
n số nguyên trong dòng lập thành hệ thặng dư đầy đủ Môđulô n. Do đó có đúng ϕ(n)
số trong hàng đó nguyên tố cùng nhau với n. Do các số đó cũng nguyên tố cùng
nhau với m nên chúng nguyên tố cùng nhau với mn. Vì có ϕ(m) dòng, mỗi dòng
chứa ϕ(n) số nguyên tố cùng nhau với mn nên ta suy ra ϕ(mn) = ϕ(m)ϕ(n).
Định lí 1.4 ([2]). Giả sử n = p
a
11 p
a
22 . . . p
a
kk là phân tích n ra thừa số
nguyên tố. Khi đó
1 1 1
Viết thuê đề tài giá rẻ trọn gói - KB Zalo/Tele : 0973.287.149
Luanvanmaster.com – Cần Kham Thảo - Kết bạn Zalo/Tele : 0973.287.149
ϕ(n) = n 1 − 1 − ... 1− .
p1 p2 pk
Viết thuê đề tài giá rẻ trọn gói - KB Zalo/Tele : 0973.287.149
Luanvanmaster.com – Cần Kham Thảo - Kết bạn Zalo/Tele : 0973.287.149
7
Chứng minh. Vì ϕ là hàm có tính chất nhân nên nếu n có phân tích
như trên thì ϕ (n) = ϕ (p
a
11 ) ϕ (p
a
22 ) . . . ϕ (p
a
kk ) . Mặt khác
ϕ pj
aj
= pj
aj
− pj
aj −1
= pj
aj
1
1 − , j = 1, 2 . . . , k
pj
Vậy
1 1
. . . pk
ak
1
ϕ (n) = p1
a1
1 − p2
a2
1− 1 −
p1 p2 pk
= p1
a1
p2
a2
. . . pk
ak
1 1 1
1 − 1 − ... 1−
p
1
p
2
p
k
1 1 1
= n 1 − 1 − ... 1− .
p1 p2 pk
Định lí 1.5 ([2]). Giả sử n là số nguyên dương. Khi đó
ϕ(d) = n,
d|n
trong đó tổng được lấy theo mọi ước của n.
Chứng minh. Tổng trên đây được lấy theo các ước của n. Ta phân chia tập hợp
các số tự nhiên từ 1 đến n thành các lớp sau đây. Lớp Cd gồm các số nguyên m, 1 ≤
m ≤ n, mà (m, n) = d. Như vậy m thuộc lớp Cd nếu và chỉ nếu d là ước chung của m,
n và (m/d, n/d) = 1. Như vậy, số phần tử của lớp Cd là số nguyên dương không vượt
quá n/d và nguyên tố cùng nhau với n/d; tức là Cd gồm ϕ(n/d) phần tử. Vì mỗi số
nguyên m từ 1 đến n thuộc một và chỉ một lớp Cd nào đó (d = (m, n)) nên n bằng
tổng của số các thành phần trong các lớp Cd, d là ước số của n. Ta có
n = ϕ n .
d|n d
Bên cạnh Phi-hàm Euler thì ta cần nghiên cứu đến lớp các hàm sinh
bởi các ước, trước tiên ta tìm hiểu về hàm tổng các ước.
Định nghĩa 1.6 ([2]). Hàm tổng các ước dương của số tự nhiên n được kí
hiệu là σ(n).
Ví dụ 1.6. σ(12) = 1 + 2 + 3 + 4 + 6 + 12 = 28.
Viết thuê đề tài giá rẻ trọn gói - KB Zalo/Tele : 0973.287.149
Luanvanmaster.com – Cần Kham Thảo - Kết bạn Zalo/Tele : 0973.287.149
Chú ý 1.2. Ta có thể biểu diễn σ(n) dưới dạng σ(n) = d.
d|n
Viết thuê đề tài giá rẻ trọn gói - KB Zalo/Tele : 0973.287.149
Luanvanmaster.com – Cần Kham Thảo - Kết bạn Zalo/Tele : 0973.287.149
8
Bổ đề 1.1 ([2]). Giả sử m, n là các số nguyên dương nguyên tố cùng nhau.
Khi đó nếu d là ước chung của mn thì tồn tại cặp duy nhất các ước dương
d1 của m và d2 của n sao cho d = d1d2. Ngược lại, nếu d1, d2 là ước
dương tương ướng của m và n thì d = d1d2 là ước dương của mn.
Chứng minh. Giả sử m, n có phân tích ra thừa số nguyên tố như sau
m = p
m
1
1
p
m
2
2
. . . p
m
s
s
, n = q1
n1
q2
n2
. . . qt
nt
.
Vì (m, n) = 1 nên tập hợp các số nguyên tố p1, p2, . . . , ps và tập hợp các
số nguyên tố q1, q2, . . . , qt không có phần tử chung. Do đó phân tích ra thừa
số của mn có dạng mn = pm
11 pm
22 . . . pm
ss .q1
n
1 q2
n
2 . . . qt
n
t . Như vậy, nếu
d là một ước chung của mn thì d = pe
1
1
pe
2
2
. . . pe
ss .q1
f1
q2
f2
. . . qt
ft
, trong
đó 0 ≤ ei ≤ mi (i = 1, 2, . . . , s) ; 0 ≤ fi ≤ ni (i = 1, 2, . . . , t) . Đặt d1 = pe
1
1
pe
2
2
.
. . pe
ss , d2 = q1
f1
q2
f2
. . . qt
ft
. Rõ ràng d = d1d2 và (d1, d2) = 1.
Ngược lại, giả sử d1 và d2 là các ước dương tương ứng của m và n khi đó d1 = p
e
11
pe
22 . . . pe
ss trong đó 0 ≤ ei ≤ mi (i = 1, 2, . . . , s) và d2 = q1
f1
q2
f2
. . . qt
ft
trong đó
0 ≤ fi ≤ ni (i = 1, 2, . . . , t) . Số nguyên d = d1d2 = pe
1
1
pe
2
2
. . . pe
ss .q1
f1
q2
f2
.
. . qt
ft
là ước của mn = pm
11 pm
22 . . . pm
ss .q1
n
1 q2
n
2 . . . qt
n
t . Vì lũy thừa của
mỗi số nguyên tố xuất hiện trong phân tích ra thừa số nguyên tố của d bé hơn
hoặc bằng lũy thừa của số nguyên tố đó trong phân tích của mn.
Bổ đề 1.2 ([2]). Giả sử p nguyên tố a nguyên dương. Khi đó:
σ(pa) = 1 + p + p
2
+ · · · + pa =
pa − 1
p − 1
τ (p
a
) = a + 1
Chứng minh. Các ước của pa
là 1, p, p2
, . . . , pa
. Do đó pa
có đúng a + 1
ước
dương. Suy ra τ (p
a
) = a + 1 và σ(p
a
) = 1 + p + p
2
+ · · · + p
a
=
pa − 1
.
p − 1
Viết thuê đề tài giá rẻ trọn gói - KB Zalo/Tele : 0973.287.149
Luanvanmaster.com – Cần Kham Thảo - Kết bạn Zalo/Tele : 0973.287.149
Định lí 1.6 ([9]). Giả sử f là hàm có tính chất nhân. Khi đó hàm F (n) = f (d)
d|n
cũng có tính chất nhân.
Viết thuê đề tài giá rẻ trọn gói - KB Zalo/Tele : 0973.287.149
Luanvanmaster.com – Cần Kham Thảo - Kết bạn Zalo/Tele : 0973.287.149
9
Chứng minh. Ta sẽ chỉ ra rằng nếu m, n là các số nguyên dương nguyên tố cùng
nhau thì F (mn) = F (m).F (n). Giả sử (m, n) = 1, ta có F (mn) = f (d). Vì
d|mn
(m, n) = 1 nên theo bổ đề 1.1, mỗi ước của mn có thể viết duy nhất dưới dạng tích
các ước d1 của m và d2 của n và d1, d2 nguyên tố cùng nhau, đồng thời mỗi cặp
ước số d1 của m và d2 của n tương ứng với ước d1.d2 của mn. Do đó ta có thể viết
F (mn) = f (d1) f (d2) = f (d1) . f (d2) = F (m) F (n) .
d1
d2
|m
|n
d1 |m d2|n
Từ định lí 1.6 và bổ đề 1.2 ta suy ra trực tiếp tính chất sau đây.
Tính chất 1.1. σ(n) là hàm nhân tính.
Tính chất 1.2. Nếu p nguyên tố thì σ(p) = p + 1.
Tính chất 1.3. Giả sử n là số nguyên dương và có khai triển chính tắc
n = pα1 pα2 . . . p
α
k thì σ(n) = p1
α
1
+1
− 1 .p2
α
2
+1
− 1 . . .pk
α
k+1 − 1
1 2 k
p1 − 1p2 − 1 pk − 1
Chứng minh. Do hàm σ là hàm nhân tính nên ta có
σ(n) = σ(p
α1
)σ(p
α2
) . . . σ(pαs
)
1 2 s
α1 α2 αk
)
= (1 + p1 + · · · + p1 ) (1 + p2 + · · · + p2 ) . . . (1 + pk + · · · + pk
= p1
α
1
+1
− 1 .p2
α
2
+1
− 1 . . .pk
α
k+1 − 1
p1 − 1 pk − 1
p2 − 1
Trong lớp hàm sinh bởi các ước thì ta cũng phải kể đến hàm số các
ước được định nghĩa như sau.
Định nghĩa 1.7 ([2]). Số các ước dương của số tự nhiên n được kí hiệu là τ (n).
Ví dụ 1.7. τ (1) = 1, τ (2) = 2, τ (12) = 6.
Định lí 1.7 ([9]). Hàm τ (n) là hàm nhân tính.
Viết thuê đề tài giá rẻ trọn gói - KB Zalo/Tele : 0973.287.149
Luanvanmaster.com – Cần Kham Thảo - Kết bạn Zalo/Tele : 0973.287.149
Chứng minh. Xét hàm f (n) ≡ 1 với mọi số tự nhiên n. Khi đó f (n) là hàm
nhân tính. Gọi d1, d2, . . . , dk là tất cả các ước dương của n (kể cả 1 và n).
Khi đó τ (n) = k = f (d1) + f (d2) + · · · + f (dk) . Vậy τ (n) là hàm nhân tính.
Viết thuê đề tài giá rẻ trọn gói - KB Zalo/Tele : 0973.287.149
Luanvanmaster.com – Cần Kham Thảo - Kết bạn Zalo/Tele : 0973.287.149
10
Định lí 1.8 ([9]). Nếu p là nguyên tố thì τ (p) = 2.
Chứng minh.Vì p nguyên tố nên p chỉ có hai ước dương là 1 và p. Do đó
τ(p) = 2.
Định lí 1.9 ([9]). Nếu số nguyên dương n có khai triển ra thừa số nguyên tố dạng
n = p
α
1
1
p
α
2
2
. . . p
α
kk thì τ (n) = (α1 + 1) (α2 + 1) . . . (αk + 1) .
Chứng minh. Vì τ (n) là hàm nhân tính nên ta có
τ (n) = τ (p
α
1
1
) τ (p
α
2
2
) . . . τ (p
α
kk ) = (α1 + 1) (α2 + 1) . . . (αk + 1) ,
bởi vì p
α
ii có αi + 1 ước dương với i = 1, 2, . . . , k.
Ngoài các hàm nhân tính kể trên thì ta cũng cần xét đến hàm nhân tính sau
Định nghĩa 1.8 (Hàm Mobius). Hàm Mobius là hàm số học xác định như sau:
1 nếu n = 1;
|
µ(n) = 0 nếu p 2 n đối với số nguyên tố p > 1;
2
n = p1p2 . . . pk ở đó , . . . , pk là các số nguyên tố phân biệt.
(−1)knếu
p
1
Ví dụ 1.8. µ(2) = −1, µ(6) = 1, µ(12) = µ(2 .3) = 0.
Định lí 1.10 ([9]). Hàm Mobius là hàm nhân tính.
Chứng minh. Giả sử m, n là những số nguyên dương sao cho gcl (m,
n) = 1. Nếu p
2
|m với moi p > 1 thì p
2
|mn và µ (m) = µ (mn) = 0 và khẳng
định trên là đúng. Bây giờ xét m = p1.p2 . . . pk và n = q1.q2 . . . qh với p1,
p2, . . . , pk và q1, q2, . . . , qh là các số nguyên tố phân biệt. Khi đó µ (m) =
(−1)
k
, µ (n) = (−1)
h
và mn = p1.p2 . . . pk.q1.q2 . . . qh. Suy ra
µ (mn) = (−1)k+h
= (−1)k
(−1)h
= µ (m) .µ (n) .
Định lí 1.11 ([9]). Cho f là một hàm số học và F (n) = f (d). Khi đó
d|n
f (n) = µ (d) F n .
Viết thuê đề tài giá rẻ trọn gói - KB Zalo/Tele : 0973.287.149
Luanvanmaster.com – Cần Kham Thảo - Kết bạn Zalo/Tele : 0973.287.149
d
d|n
Viết thuê đề tài giá rẻ trọn gói - KB Zalo/Tele : 0973.287.149
Luanvanmaster.com – Cần Kham Thảo - Kết bạn Zalo/Tele : 0973.287.149
11
Chứng minh. Ta có
n
c| c|
n µ(d) f (c)
d|n µ (d) F d
=
d|n
µ (d)
n f (c) = d|n
d d
d|
= c|n n µ (d) f (c) =
c
d|
c|n f (c) n µ (d) = f (n) .
c
Từ
n
> 1 ta có n µ(d) = 0. Thực tế các tập (d, c) |d|n, c|
n
và (d, c) |c|n, d|
n
c d d
d| c
là bằng nhau.
1.1.2 Hàm nhân tính mạnh
Định nghĩa 1.9 ([9]). Một hàm số học f được gọi là nhân tính mạnh nếu với
mọi m, n (không nhất thiết nguyên tố cùng nhau), ta có đẳng thức
f (mn) = f (m)f (n).
Những ví dụ đơn giản nhất về hàm nhân tính mạnh là f (n) = n và f (n) = 1.
1.2 Hàm số xác định trên tập các số nguyên
1.2.1 Hàm cộng tính trên tập các số nguyên
Định nghĩa 1.10 ([9]). Một hàm số f (x) xác định trên tập số nguyên được
gọi là hàm cộng tính nếu với mọi m, n ta có đẳng thức
f (m + n) = f (m) + f (n).
1.2.2 Hàm nhân tính trên tập các số nguyên
Định nghĩa 1.11 ([9]). Một hàm số f (x) xác định trên tập số nguyên được
gọi là hàm nhân tính nếu với mọi m, n ta có đẳng thức
Viết thuê đề tài giá rẻ trọn gói - KB Zalo/Tele : 0973.287.149
Luanvanmaster.com – Cần Kham Thảo - Kết bạn Zalo/Tele : 0973.287.149
f (mn) = f (m)f (n).
Viết thuê đề tài giá rẻ trọn gói - KB Zalo/Tele : 0973.287.149
Luanvanmaster.com – Cần Kham Thảo - Kết bạn Zalo/Tele : 0973.287.149
12
1.2.3 Lớp hàm tuần hoàn, phản tuần hoàn cộng tính, nhân tính
Định nghĩa 1.12 ([5]). Cho hàm số f (x) xác định trên tập số nguyên. Hàm f (x)
được gọi là hàm tuần hoàn trên Z nếu tồn tại số nguyên dương a sao cho
∀x ∈ Z ta đều có x ± a ∈ Z
f (x + a) = f (x) , ∀x ∈ Z
a được gọi là chu kỳ của hàm tuần hoàn f (x). Chu kỳ nhỏ nhất (nếu có) trong
các chu kỳ của f (x) được gọi là chu kỳ cơ sở của hàm tuần hoàn f (x).
Ví dụ 1.9. Cho cặp hàm f (x), g(x) xác định trên tập số nguyên, tuần hoàn trên
Z và có chu kỳ lần lượt là a và b (a, b ∈ Z+
) với
a
∈ Q+
. Chứng minh rằng
b
Lời giải. Theo giả thiết tồn tại m, n ∈ Z
+
, (m, n) = 1 sao cho
a
b =
m
n. Đặt
T = na = mb. Khi đó
T
F (x + T ) = f (x + na) + g(x + mb) = f (x) + g(x) = F (x)
G(x + T ) = f (x + na)g(x + mb) = f (x)g(x) = G(x)
Hơn nữa, dễ thấy ∀x ∈ Z thì x ± T ∈ Z. Vậy F (x) và G(x) là những hàm
tuần hoàn trên Z.
Ví dụ 1.10. Xét hàm số f (x) = sin
2
3
π
x xác định trên Z. Chứng minh
rằng f (x) là hàm tuần hoàn với chu kỳ T = 3.
Lời giải. Ta có ∀x ∈ Z thì x ± 3 ∈ Z và
2π 2π
f (x + 3) = sin 3 (x + 3) = sin 3 x + 2π = f (x).
Định nghĩa 1.13 ([5]). Cho hàm số f (x) xác định trên tập số nguyên. Hàm f (x)
được gọi là hàm phản tuần hoàn trên Z nếu tồn tại số nguyên dương a sao cho
Viết thuê đề tài giá rẻ trọn gói - KB Zalo/Tele : 0973.287.149
Luanvanmaster.com – Cần Kham Thảo - Kết bạn Zalo/Tele : 0973.287.149
∀x ∈ Z ta đều có x ± a ∈ Z
f (x + a) = −f (x) , ∀x ∈ Z
Viết thuê đề tài giá rẻ trọn gói - KB Zalo/Tele : 0973.287.149
Luanvanmaster.com – Cần Kham Thảo - Kết bạn Zalo/Tele : 0973.287.149
13
a được gọi là chu kỳ của hàm phản tuần hoàn f (x). Chu kỳ nhỏ nhất (nếu có)
trong các chu kỳ của f (x) được gọi là chu kỳ cơ sở của hàm phản tuần hoàn f (x).
Ví dụ 1.11. Chứng minh rằng mọi hàm phản tuần hoàn xác định trên tập số
nguyên cũng là hàm tuần hoàn xác định trên tập số nguyên.
Lời giải. Theo giả thiết tồn tại số nguyên dương b sao cho ∀x
∈ Z thì x ± b ∈ Z và f (x + b) = −f (x), ∀x ∈ Z.
Suy ra ∀x ∈ Z thì x ± 2b ∈ Z và
f (x + 2b) = f (x + b + b) = −f (x + b) = f (x), ∀x ∈ Z.
Vậy f (x) là hàm tuần hoàn với chu kỳ 2b trên Z.
Ví dụ 1.12. Xét hàm số f (x) = sin
π
3 x xác định trên Z. Chứng minh rằng
f (x) là hàm phản tuần hoàn với chu kỳ T = 3.
Lời giải. Ta có ∀x ∈ Z thì x ± 3 ∈ Z và
π π
f (x + 3) = sin 3 (x + 3) = sin 3 x + π = −f (x).
Định nghĩa 1.14 ([5]). Hàm f (x) xác định trên tập số nguyên được gọi là
tuần hoàn nhân tính chu kỳ nguyên dương a (a ∈/ {−1, 0, 1}) trên Z nếu
∀x ∈ Z ⇒ a
±1
x ∈ Z
f (ax) = f (x) , ∀x ∈ Z
Ví dụ 1.13. Xét hàm số f (x) = sin (2πlog2x) xác định trên Z
+
. Khi đó f (x) là
hàm tuần hoàn nhân tính chu kỳ 2. Thật vậy, ta có ∀x ∈ Z
+
thì 2
±1
x ∈ Z
+
và
f (2x) = sin (2πlog2(2x))
Viết thuê đề tài giá rẻ trọn gói - KB Zalo/Tele : 0973.287.149
Luanvanmaster.com – Cần Kham Thảo - Kết bạn Zalo/Tele : 0973.287.149
= sin (2π (1 + log2x))
= sin (2πlog2x) = f (x), ∀x ∈ Z+
.
Viết thuê đề tài giá rẻ trọn gói - KB Zalo/Tele : 0973.287.149
Luanvanmaster.com – Cần Kham Thảo - Kết bạn Zalo/Tele : 0973.287.149
14
Định nghĩa 1.15 ([5]). Hàm f (x) xác định trên tập số nguyên được gọi là
hàm phản tuần hoàn nhân tính chu kỳ dương a (a ∈/ {−1, 0, 1}) trên Z nếu
∀x ∈ Z ⇒ a
±1
x ∈ Z
f (ax) = −f (x) , ∀x ∈ Z
Ví dụ 1.14. Xét hàm số f (x) = sin (πlog2x) xác định trên Z
+
. Khi đó f (x) là hàm
phản tuần hoàn nhân tính chu kỳ 2. Thật vậy, ta có ∀x ∈ Z
+
thì 2
±1
x ∈ Z
+
và
f (2x) = sin (πlog2(2x))
= sin (π (1 + log2x))
= − sin (πlog2x) = −f (x), ∀x ∈ Z+
.
1.3 Một số bài tập áp dụng
Bài toán 1.1. Tìm tất cả các số tự nhiên n có tính chất n chia hết cho ϕ(n).
Lời giải. Ta xét các trường hợp sau.
- Nếu n = 1 thì ϕ(n)|n.
- Nếu n > 1, giả sử n có phân tích thành thừa số nguyên tố là n = p
k
11 p
k
22 . . . p
k
ii .
Khi đó, ta có
1 1 1
ϕ(n) = n 1 − 1 − ... 1− .
p1 p2 pi
Từ điều kiện n = xϕ(n) ta suy ra
p1p2 . . . pi = x (p1 − 1) (p2 − 1) . . . (pi − 1) .
Điều này có nghĩa là phải có pj nào đó bằng 2 (vì nếu ngược lại thì sẽ dẫn
đến vô lí vì vế trái là số lẻ, trong khi vế phải là số chẵn).
Do p2, . . . , pi khác 2 nên từ trên suy ra rằng n có nhiều nhất một ước
nguyên tố lẻ, chẳng hạn p2.
Đặt p2 = 2y + 1. Ta có 2p2 = x(2y). Do p2 là nguyên tố nên x = p2, y = 1.
Như vậy p2 = 3 và n có dạng n = 2
k
2
m
với k ≥ 1 và m ≥ 0.
Viết thuê đề tài giá rẻ trọn gói - KB Zalo/Tele : 0973.287.149
Luanvanmaster.com – Cần Kham Thảo - Kết bạn Zalo/Tele : 0973.287.149
Thử lại thấy n = 2
k
2
m
với k ≥ 1, m ≥ 0 thỏa mãn.
Viết thuê đề tài giá rẻ trọn gói - KB Zalo/Tele : 0973.287.149
Luanvanmaster.com – Cần Kham Thảo - Kết bạn Zalo/Tele : 0973.287.149
15
Bài toán 1.2. Chứng minh rằng ϕ(n) là chẵn nếu n = 1 hoặc n = 2 và lẻ nếu
với mọi n ≥ 3.
Lời giải. Ta xét các trường hợp sau.
- Với n = 1 hoặc n = 2 ta có ϕ(1) = ϕ(2) = 1 lẻ.
- Với n ≥ 3 và n = p
α
11 p
α
22 . . . p
α
rr . ta có
ϕ(n) = p
α
1
1−1
p
α
2
2−1
. . . p
α
rr
−1
(p1 − 1) (p2 − 1) . . . (pr − 1) .
Vì ít nhất một trong các số hạng p1, p2, . . . , pr, p1 − 1, p2 − 1, . . . , pr − 1
là chẵn nên ϕ(n) là chẵn.
Bài toán 1.3 (Canada 1999). Tìm tất cả các số nguyên dương n
sao cho n = τ 2
(n).
Lời giải. Giả sử n là số nguyên dương thỏa mãn yêu cầu bài toán.
k
Do n = τ 2
(n) là một số chính phương nên ta có thể đặt n =
i=1
τ (n) = k (2αi + 1)
i
i=1
k
2α +1
i=1 pi
αi
= 1
Áp dụng bất đẳng thức Bernoulli ta có
p
α
i
i
≥ (pi − 1) αi + 1 > 2αi + 1.
Với mọi ước số nguyên tố pi ≥ 5 của n.
Mặt khác, ta có 3
α
≥ 2α + 1 và dấu bằng xảy ra khi và chỉ khi α ∈ {0, 1}.
Suy ra n = 1, n = 9 thỏa mãn yêu cầu bài toán.
Bài toán 1.4. Tìm các số nguyên dương n sao cho σ(n) = 12.
Lời giải. Để giải quyết tốt bài toán này ta cần chọn lựa các số nguyên tố
và số mũ thích hợp sao cho:
(1 + p1 + · · · + p
α
11 ) (1 + p2 + · · · + p
α
22 ) . . . (1 + pk + · · · + p
α
kk ) = σ(n).
Liệt kê các số có dạng 1 + pk + p
2
k + · · · + p
α
kk từ nhỏ đến lớn
p2
i
α
i . Khi đó
Viết thuê đề tài giá rẻ trọn gói - KB Zalo/Tele : 0973.287.149
Luanvanmaster.com – Cần Kham Thảo - Kết bạn Zalo/Tele : 0973.287.149
1+2;1+3;1+5;1+2+2
2
;1+7;1+3+3
2
;1+2+2
2
+2
3
,...
Viết thuê đề tài giá rẻ trọn gói - KB Zalo/Tele : 0973.287.149
Luanvanmaster.com – Cần Kham Thảo - Kết bạn Zalo/Tele : 0973.287.149
16
Với σ(n) = 12 ta thấy chỉ có thể viết thành 12 = (1 + 2)(1 + 3) và số cần tìm là
n = 6.
Bài toán 1.5. Với giá trị nào của n thì σ(n) là số lẻ
Lời giải. Cần chọn các ước số nguyên tố và số mũ thích hợp để σ(n) là số lẻ. Ta có
σ(n) = (1 + p1 + · · · + pα
1
1
) (1 + p2 + · · · + pα
2
2
) . . . (1 + pk + · · · + pα
kk ) .
Khi đó nếu có ước nguyên tố nào đó là 2 thì vẫn thỏa mãn; nếu n có ước nguyên
tố lẻ thì trong tổng trên chúng phải xuất hiện chẵn lần. Do đó số cần tìm có dạng
n = 2
k
m
2
với m là số chính phương lẻ.
Bài toán 1.6. Chứng minh rằng điều kiện cần và đủ để n là một hợp số là:
σ(n) +
1
4 >
√
n +
1
2.
Lời giải. Trước hết, ta có
σ(n) +
1
4 >
√
n +
1
2 ⇔σ(n) +
1
4 > n +
√
n +
1
4 ⇔σ(n) > n +
√
n.
Phần thuận
Giả sử n là hợp số. Khi đó ngoài các ước 1 và n thì n còn ít nhất một
ước là d với 1 < d < n. Lúc này
n
d cũng là một ước của n ( dễ thấy 1 <
n
d < n). Xét hai trường hợp sau:
- Nếu
n
d = d thì sẽ có ít nhất 4 ước là 1, n, d,
n
d. Khi đó
σ(n) ≥ 1 + n + d +
n
≥ 1 + n + 2
n √ √
d. = 1 + n + 2 n > n + n.
d d
- Nếu
n
d = d tức d =
√
n thì lúc này n có ít nhất 3 ước là 1,
√
n, n. Vì thế
σ(n) ≥ 1 + n +
√
n > n +
√
n.
Như vậy khi n là hợp số thì σ(n) > n +
√
n.
Phần đảo
Giả sử σ(n) > n +
√
n. Vì σ(1) = 1 < 1 +
√
1 vô lý, cho nên n = 1. Hơn nữa n
Viết thuê đề tài giá rẻ trọn gói - KB Zalo/Tele : 0973.287.149
Luanvanmaster.com – Cần Kham Thảo - Kết bạn Zalo/Tele : 0973.287.149
không thể là số nguyên tố. Thật vậy, nếu p là số nguyên tố thì σ(p) = p + 1
< p +
√
p ( chú ý p ≥ 2 ), lại là điều vô lý.
Viết thuê đề tài giá rẻ trọn gói - KB Zalo/Tele : 0973.287.149
Luanvanmaster.com – Cần Kham Thảo - Kết bạn Zalo/Tele : 0973.287.149
17
Tóm lại, nếu n thỏa mãn bất đẳng thức σ(n) > n +
√
n. thì n phải khác 1
và không phải số nguyên tố. Điều này đồng nghĩa với n phải là hợp số.
Vậy ta hoàn tất chứng minh.
Bài toán 1.7. Ký hiệu d(n) là trung bình cộng của tất cả các ước số của n (
kể cả 1 và n). Chứng minh rằng với mọi n ta có bất đẳng thức
√ n + 1
n ≤ d(n) ≤ .
2
Lời giải. Giả sử 1 = d1 < d2 < · · · < dk = n là tất cả các ước của n.
Như vậy 1 ≤ di ≤ n với i = 1, 2, . . . , n.
Theo định nghĩa ta có d(n) = d1 + d2 + · · · + dk .
n n n
k
Để ý rằng > > ··· > >
n
và chúng cũng là tất cả các ước của n. Do
d1 d2 dk−1 dk
đó
n n n n
d1 = , d2 = , . . . , dk−1 = , dk = .
dk dk−1 d2 d1
Vì thế ta có
d1dk = d2dk−1 = · · · = d1dk−i+1 = n với i = 1, 2, . . . , k − 1. (1.1)
Ta có
2(d1 + d2 + · · · + dk)
d1 + d2 +d2 + dk−1 +... dk−1 + d2 +dk + d1
d(n) = = 2 2 2 2 .
k
2k
Áp dụng bất đẳng thức AM - GM ta có:
√ +
√
+···+
√ +
√
.
d(n) d1dk d2dk−1 dk−1d2 dkd1 (1.2)
≥ k
√
Từ (1.1) và (1.2) ta suy ra d(n) ≥
k n
= √
n. Chứng minh xong bất đẳng
thức thứ k
nhất. Ta sẽ chứng minh bất đẳng thức còn lại.
di ≥ 1 với mọi i = 1, 2, . . . , k;
Nên
Viết thuê đề tài giá rẻ trọn gói - KB Zalo/Tele : 0973.287.149
Luanvanmaster.com – Cần Kham Thảo - Kết bạn Zalo/Tele : 0973.287.149
0 ≤ (di − 1) (dk−i+1 − 1) = didk−i+1 + 1 − di − dk−i+1. (1.3)
Viết thuê đề tài giá rẻ trọn gói - KB Zalo/Tele : 0973.287.149
Luanvanmaster.com – Cần Kham Thảo - Kết bạn Zalo/Tele : 0973.287.149
18
Từ (1.1) và (1.3) suy ra
n + 1 − (di + dk−i+1) ≥ 0, với mọi i = 1, 2, . . . , k (1.4)
Cộng vế theo vế k bất đẳng thức dạng (1.4) ta được:
k 1 k n + 1
2
i=1
d
i
≤ k (n + 1) ⇔
i=1
d
i
≤
k 2
Chứng minh xong bất đẳng thức còn lại. Vậy ta hoàn tất việc chứng minh
bất đẳng thức kép ở trên.
Bài toán 1.8 (Korean MO 1998). Cho n là một số nguyên dương, ký hiệu ψ(n) là
số các thừa số nguyên tố của n. Chứng minh rằng nếu ϕ(n) chia hết cho n − 1 và
ψ(n) ≤ 3 thì n là số nguyên tố.
Lời giải. Để ý rằng với số nguyên tố p, nếu p2
|n thì p|ϕ(n) nhưng p không
là ước của n − 1, điều này mâu thuẫn. Vì vậy chúng ta chỉ cần chứng tỏ
rằng n = pq, n = pqr với các số nguyên tố p < q < r.
Trước hết, giả sử n = pq, vì vậy
(p − 1)(q − 1)|pq − 1
Chú ý rằng q ≥ 3 suy ra vế trái là số chẵn, cho nên vế phải cũng vậy và
p, q là các số lẻ. Xét trường hợp p = 3, q = 5. Khi đó
pq − 1
< 2. (1.5)
(p − 1)(q − 1)
Vế trái của (1.5) là một hàm tăng với từng biến và luôn lơn hơn 1, vì vậy nó không
thể là số nguyên. Đây là điều mâu thuẫn.
Bây giờ, xét n = pqr. Như trên p, q, r là lẻ ; nếu p = 3, q = 5 và r = 11 thì
pqr − 1 < 2. (1.6)
(p − 1)(q − 1)(r − 1)
Viết thuê đề tài giá rẻ trọn gói - KB Zalo/Tele : 0973.287.149
Luanvanmaster.com – Cần Kham Thảo - Kết bạn Zalo/Tele : 0973.287.149
Vế trái của (1.6) là một hàm tăng với từng biến và luôn lơn hơn 1, vì
vậy nó không thể là số nguyên. Đây là điều mâu thuẫn.
Viết thuê đề tài giá rẻ trọn gói - KB Zalo/Tele : 0973.287.149
Luanvanmaster.com – Cần Kham Thảo - Kết bạn Zalo/Tele : 0973.287.149
19
Điều này giúp loại bỏ các trường hợp ngoại trừ p = 3, q = 5 và r = 7 ta có
pqr − 1 < 3. (1.7)
(p − 1)(q − 1)(r − 1)
Vì vế trái của (1.7) chỉ nhận giá trị nguyên nên
pqr − 1 = 2.
(p − 1)(q − 1)(r − 1)
Để ý rằng
15r − 1
= 2 cho ta r = 15 không phải là số nguyên tố và ta loại
bỏ 8(r − 1)
tất cả các trường hợp.
Viết thuê đề tài giá rẻ trọn gói - KB Zalo/Tele : 0973.287.149
Luanvanmaster.com – Cần Kham Thảo - Kết bạn Zalo/Tele : 0973.287.149
20
Chương 2
Các phương trình hàm số học
Trong chương này sẽ trình bày về hàm chuyển đổi các phép tính số
học, các dạng toán xác định dãy số liên quan và các bài tập áp dụng.
2.1 Hàm chuyển đổi các phép tính số học
Trong mục này, ta giải các bài toán xác định các hàm chuyển đổi các phép tính
số học đơn giản của đối số sang phép tính đối với các giá trị hàm tương ứng. Ta sẽ
giải quyết các bài toán trên lớp hàm xác định trên tập số nguyên dương.
2.1.1 Hàm chuyển đổi phép cộng thành phép cộng
Bài toán 2.1. Xác định tất cả các hàm số f : N∗
→ R thỏa mãn
f (x + y) = f (x) + f (y), ∀x, y ∈ N
∗
. (2.1)
Lời giải. Từ (2.1) suy ra với x = y thì
f (2x) = 2f (x), ∀x ∈ N
∗
Giả sử với k nguyên dương, f (kx) = kf (x), ∀x ∈ N
∗
. Khi đó
f ((k + 1) x) = f (kx) + f (x)
= kf (x) + f (x)
= (k + 1) f (x) , ∀x ∈ N
∗
, ∀k ∈ N
∗
.
Từ đó, theo nguyên lí quy nạp, ta có
Viết thuê đề tài giá rẻ trọn gói - KB Zalo/Tele : 0973.287.149
Luanvanmaster.com – Cần Kham Thảo - Kết bạn Zalo/Tele : 0973.287.149
f (nx) = nf (x), ∀x ∈ N
∗
, ∀n ∈ N
∗
Viết thuê đề tài giá rẻ trọn gói - KB Zalo/Tele : 0973.287.149
Luanvanmaster.com – Cần Kham Thảo - Kết bạn Zalo/Tele : 0973.287.149
21
Tóm lại ta có
f (nx) = nf (x), ∀x ∈ N
∗
∀n ∈ N
∗
.
Trong quan hệ trên cho x = 1 ta được f (n) = cn, ∀n ∈ N∗
(c = f (1) là hằng số).
Thử lại ta thấy hàm này thỏa mản bài ra.
Nhận xét.
- Kết quả của bài toán vẫn đúng khi ta mở rộng tập nguồn và tập đích lên tập Z.
- Kết quả của bài toán rất quan trọng, được dùng để giải nhiều phương
trình hàm trên Z.
2.1.2 Hàm chuyển đổi phép cộng thành phép nhân
Bài toán 2.2. Xác định tất cả các hàm số f : N
∗
→ R thỏa mãn
f (x + y) = f (x)f (y), ∀x, y ∈ N∗
. (2.2)
Lời giải. Với x chẵn ta thu được
f (x) = f (2m) = [f (m)]
2
≥ 0, ∀m ∈ N
∗
.
Xét các trường hợp:
- Nếu f (1) = 0, thì f (n + 1) = f (1)f (n) = 0 nên f (x) ≡ 0 và hàm này thỏa
mãn điều kiện (2.2) nên là một nghiệm của bài toán.
- Xét trường hợp f (1) = a > 0. Khi đó f (2) = [f (1)2
] = a2
> 0 và f (3) =
f (1)f (2 + 1) = a
3
> 0. Bằng phương pháp quy nạp, ta chứng minh được f
(n) = a
n
với mọi n ∈ N
∗
.
- Xét trường hợp f (1) = b < 0. Khi đó f (2) = [f (1)
2
] = b
2
> 0 và f (3) =
f (1)f (2 + 1) = b
3
< 0. Bằng phương pháp quy nạp, ta chứng minh được f
(n) = b
n
với mọi n ∈ N
∗
.
Kết luận.
Viết thuê đề tài giá rẻ trọn gói - KB Zalo/Tele : 0973.287.149
Luanvanmaster.com – Cần Kham Thảo - Kết bạn Zalo/Tele : 0973.287.149
Vậy f (x) ≡ 0 và f (x) = a
x
, với a = 0.
Viết thuê đề tài giá rẻ trọn gói - KB Zalo/Tele : 0973.287.149
Luanvanmaster.com – Cần Kham Thảo - Kết bạn Zalo/Tele : 0973.287.149
22
2.1.3 Hàm chuyển đổi phép nhân thành phép cộng
Bài toán 2.3. Xác định tất cả các hàm số f : N
∗
→ R và thỏa mãn
f (xy) = f (x) + f (y), ∀x, y ∈ N
∗
. (2.3)
Lời giải. Từ (2.3), ta thu được
ef (xy) = ef (x)ef (y), ∀ x, y ∈ N∗ . (2.4)
Đăt ef (t)
= g(t), thì g(x) > 0, ∀t ∈ N∗
và (2.4) có dạng
g(xy) = g(x)g(y), ∀x, y ∈ N
∗
.
Vậy g(t) là hàm dương nhân tính mạnh trên N
∗
. Suy ra f (x) = ln g(t) trong
đó g(t) là hàm dương nhân tính mạnh tùy ý trên N∗
.
2.2 Các dạng toán xác định dãy số liên quan
Bài toán phương trình hàm trên N trên một phương diện nào đó, có thể
coi là bài toán dãy số. Vì vậy, dưới đây chúng ta sẽ xét một số bài toán xác
định dãy số liên quan.
Bài toán 2.4 (IMO 1983). Chứng minh hoặc phủ định mệnh đề sau: Từ tập
hợp 10
5
số nguyên dương đầu tiên luôn có thể chọn ra một tập con gồm
1983 số sao cho không có ba số nào lập thành cấp số cộng.
Lời giải. Ta chứng minh mệnh đề tổng quát: Từ 3
n
số tự nhiên đầu tiên luôn có
thể chọn ra 2n số sao cho không có ba số nào lập thành một cấp số cộng. Thật
vậy xét trong hệ đếm cơ số 3 tập hợp tất cả các số có ≤ n chữ số. Chọn các số
mà trong biểu diễn tam phân của nó chỉ chứa chữ số 2 và chữ số 0. Khi đó có 2
n
số như vậy và không có ba số nào trong chúng lập thành cấp số cộng.
Bài toán 2.5 (Singapore 1995). Cho dãy số {fn} xác định bởi
Viết thuê đề tài giá rẻ trọn gói - KB Zalo/Tele : 0973.287.149
Luanvanmaster.com – Cần Kham Thảo - Kết bạn Zalo/Tele : 0973.287.149
f1 = 1, f2n = fn, và f2n+1 = f2n + 1.
Viết thuê đề tài giá rẻ trọn gói - KB Zalo/Tele : 0973.287.149
Luanvanmaster.com – Cần Kham Thảo - Kết bạn Zalo/Tele : 0973.287.149
23
i) Tính M = max {f1, f2, . . . , f1994} .
ii) Tìm tất cả các giá trị của n, 1 ≤ n ≤ 1994 sao cho fn = M.
Lời giải. Có thể dùng quy nạp để chứng minh rằng fn là số tất cả các chữ
số 1 trong biểu diễn nhị phân của số n.
i) Tồn tại nhiều nhất là 10 chữ số 1 trong biểu diễn nhị phân của một số
nếu số đó bé hơn hoặc bằng 1994 = 11111001010(2) suy ra M = 10.
ii) Với mọi số tự nhiên n ≤ 1994, ta có fn = 10 nếu và chỉ nếu n là một trong
các số
1023 = 1111111111(2)
1535 = 10111111111(2)
1791 = 11011111111(2)
1919 = 11101111111(2)
1983 = 11110111111(2).
Bài toán 2.6. Cho dãy số {an} xác định bởi
≤ 0
0 a < 1, an = 2an−1 nếu 2an−1 < 1 (2.5)
2an−1 − 1 nếu 2an−1 ≥ 1
Hỏi có bao nhiêu giá trị a0 sao cho a5 = a0.
Lời giải. Khi tính an theo an−1 từ quan hệ (2.5) ta có thể lựa chọn một
trong hai công thức. Tất nhiên, với a0 đã chọn rồi thì tất cả các bước tiếp
theo đều xác định một cách duy nhất. Tuy nhiên, ta có thể chọn a0 như thế
nào đó để sau đó các công thức tính theo đúng kịch bản đã cho. Có 2
5
=
32 kịch bản như vậy. Ví dụ với kịch bản (1, 1, 2, 1, 2) ta có
x1 = 2x0, x2 = 2x1 = 4x0, x3 = 2x2 − x1 = 8x0 − x1,
x4 = 2x3 = 16x0 − 2, x5 = 2x4 − 1 = 32x0 − 3.
Viết thuê đề tài giá rẻ trọn gói - KB Zalo/Tele : 0973.287.149
Luanvanmaster.com – Cần Kham Thảo - Kết bạn Zalo/Tele : 0973.287.149
Giải phương trình x5 = x0 ta được x0 = 31
3
. Tất nhiên để có một lời giải hoàn chỉnh, ta
cần phải lập luận chặt chẽ để thấy rằng các x0 thu được là khác nhau và với mỗi x0 thu
được, dãy số sẽ đi đúng như kịch bản đã định. Tuy nhiên, phân tích này hướng
Viết thuê đề tài giá rẻ trọn gói - KB Zalo/Tele : 0973.287.149
Luanvanmaster.com – Cần Kham Thảo - Kết bạn Zalo/Tele : 0973.287.149
24
chúng ta tới hệ nhị phân. Và ta có lời giải đẹp như sau:
Giả sử a0 = (0d1d2d3 . . . )2 thì a1 = (0d2d3d4 . . . )2. Thật vậy
- Nếu 2a0 < 1 thì d1 = 0 và a1 = 2a0 = (0d2d3d4 . . . )2.
- Nếu 2a0 ≥ 1 thì d1 = 1 và a1 = 2a0 − 1 = (0d2d3d4 . . . )2.
Hoàn toàn tương tự, ta có a2 = (0d3d4d5 . . . )2, . . . , a5 = (0d6d7d8 . . . )2.
Như vậy a5 = a0 khi và chỉ khi a0 là phân số nhị phân tuần hoàn chu kỳ 5.
Có 2
5
= 32 chu kỳ tuần hoàn như vậy, trong đó chu kỳ (11111)2 cho chúng
ta a0 = 1 (loại).
Vậy có tất cả 31 giá trị a0 thỏa mãn (2.5). Đó là 0, (00000), 0, (00001), . . . , 0, (11110).
Tính sang hệ thập phân đó là các giá trị 0, 1 , 2 ,...,30 .
31
31 31
Bài toán 2.7. Xác định số hạng tổng quát của dãy {xn}, biết x1 = a và
xm+n = xm + xn + mn, ∀m, n ∈ N∗
. (2.6)
Lời giải. Giả sử tồn tại dãy số {xn} thỏa mãn điều kiện bài toán. Từ phương trình
(2.6) suy ra xn+1 − xn = a + n (2.7)
x1 = a
Phương trình xn+1 − xn = a + n là một phương trình sai phân tuyến tính không
thuần nhất, cấp một. Vì phương trình đặc trưng là λ = 1 nên ta có nghiệm tổng
quát của phương trình thuần nhất xn+1 − xn = 0 như sau: xn = c. Nghiệm
riêng của (2.7) có dạng x∗
n = n(dn + e). Thay x∗
n vào (2.7), ta được
(n + 1) [d(n + 1) + e] − n (dn + e) = a + n
1
⇔ ⇐
d = 2
2dn + d + e = a + n 1
1 1
Suy ra xn
∗
=
e = a − 2
n2
+ a − n. Vì xn = xn + xn
∗
nên nghiệm phương trình (2.7) là
2 2
1 1
Viết thuê đề tài giá rẻ trọn gói - KB Zalo/Tele : 0973.287.149
Luanvanmaster.com – Cần Kham Thảo - Kết bạn Zalo/Tele : 0973.287.149
xn = c + n
2
+ a − n.
2 2
Viết thuê đề tài giá rẻ trọn gói - KB Zalo/Tele : 0973.287.149
Luanvanmaster.com – Cần Kham Thảo - Kết bạn Zalo/Tele : 0973.287.149
25
Vì x1 = a nên ta có c = 0. Suy ra
1 1
xn = n
2
+ a − n.
2 2
1
n
2
1
Thử lại, ta thấy nghiệm xn = + a − n. Thỏa mãn điều kiện bài toán.
2 2
Bài toán 2.8. Tồn tại hay không một dãy số {xn}, thỏa mãn điều kiện
xm+n = xm + xn + m + n, ∀m, n ∈ N
∗
. (2.8)
Lời giải. Giả sử tồn tại dãy số {xn} thỏa mãn điều kiện bài toán. Từ
phương trình (2.8) cho m = 1 suy ra
xn+1 = xn + x1 + n + 1,
hay
xn+1 − xn = a + n, với a = x1 + 1. (2.9)
Phương trình xn+1 − xn = a + n là một phương trình sai phân tuyến tính không
thuần nhất, cấp một. Vì phương trình đặc trưng là λ = 1 nên ta có nghiệm tổng
quát của phương trình thuần nhất xn+1 − xn = 0 là xn = c. Nghiệm riêng của
(2.9) có dạng x∗
n = n(dn + e), (d = 0). Thay x∗
n vào (2.9) ta được
(n + 1) [d(n + 1) + e] − n(dn + e) = a + n
⇔2dn + d + e = a + n
1
1
⇔
d = 2
e = a − 2
Suy ra xn
∗
=
1
n
2
+ a −
1
n. Vì xn = xn + xn
∗
nên ta có nghiệm của (2.9) là
2 2
1
n
2
+
1
(2.10)
xn = c + a − n.
2 2
1
n
2
+ a −
1
Vì x1 = a − 1 nên từ (2.10) ta có c = −1. Vì thế xn = n − 1.
2 2
Viết thuê đề tài giá rẻ trọn gói - KB Zalo/Tele : 0973.287.149
Luanvanmaster.com – Cần Kham Thảo - Kết bạn Zalo/Tele : 0973.287.149
Thử lại ta thấy nghiệm này không thỏa mãn bài ra.
Vậy không tồn tại dãy số thỏa mãn bài ra.
Viết thuê đề tài giá rẻ trọn gói - KB Zalo/Tele : 0973.287.149
Luanvanmaster.com – Cần Kham Thảo - Kết bạn Zalo/Tele : 0973.287.149
26
Bài toán 2.9. Xác định dãy số dương {xn}, thỏa mãn điều kiện
xmn = xmxn, ∀m, n ∈ N
∗
. (2.11)
Lời giải. Giả sử tồn tại dãy số dương {xn} thỏa mãn (2.11). Ta có
x1.n = x1xn ⇒ x1 = 1. Nhận xét rằng. Nếu n = pk
với p là số nguyên tố thì xn
= xpk = (xp)
k
. Ta chứng minh nhận xét trên bằng quy nạp. Với k = 1 ta có xn
= x
1
p = (xp)
1
. Giả sử nhận xét đúng với k = q, (q ≥ 1). Khi đó, với n = p
k+1
,
ta có
xn = xpk+1 = xpk xp = (xp)k
xp = (xp)k+1
.
Do đó với n = pk
thì xn = xpk = (xp)k
. Từ đây suy ra, nếu n = pm
11 . . . pm
ss thì
xn = (xp1 )m1
. . . (xps )m
s . Vậy xp có thể nhận giá trị dương tùy ý khi p là một
số nguyên tố. Do vậy, ta kết luận như sau: xp có thể nhận giá trị dương tùy ý
khi p là một số nguyên tố và xn = (xp1 )m1
. . . (xps )ms
. khi n = pm
11 . . . pm
ss .
Bài toán 2.10 (IMO 2009). Giả sử s1, s2, · · · , sn, · · · là một dãy số tăng
nghiêm ngặt các số nguyên dương thỏa mãn: Các dãy con
ss1 , ss2 , · · · , ssn , · · · và ss1+1, ss2+1, · · · , ssn+1, · · ·
là các cấp số cộng. Chứng minh rằng bản thân dãy s1, s2, · · · , sn, · · ·
cũng là cấp số cộng.
Lời giải. Để dễ sử lí, ta phát biểu bài toán dưới dạng.
Cho f : N
∗
→ N
∗
là hàm tăng thực sự và thỏa mãn: Các dãy số (f (f
(n))) và f (f (n) + 1) là cấp số cộng. Chứng minh rằng dãy (f (n)) cũng là cấp
số cộng. Từ giả thiết suy ra f (f (n)) = an + b và f (f (n)+) = cn + d.
Với a, c là những số nguyên dương, b, d là các số nguyên, ta có
f (f (n)) < f (f (n) + 1) ≤ f (f (n + 1)).
Suy ra an + b < cn + d ≤ a(n + 1) + b hay a + n
b
< c + n
d
≤ a +
a +
n
b
. Bằng cách cho n → +∞ ta được a = c. Tóm lại ta có
Viết thuê đề tài giá rẻ trọn gói - KB Zalo/Tele : 0973.287.149
Luanvanmaster.com – Cần Kham Thảo - Kết bạn Zalo/Tele : 0973.287.149
f (f (n)) = an + b, f (f (n)+) = an + c, ∀n ∈ N
∗
.
Viết thuê đề tài giá rẻ trọn gói - KB Zalo/Tele : 0973.287.149
Luanvanmaster.com – Cần Kham Thảo - Kết bạn Zalo/Tele : 0973.287.149
27
Dễ dàng chứng minh được
f (an + b) = af (n) + b và f (an + b + 1) = af (n) + c.
Ta có 0 < f (n + 1) − f (n) ≤ a, ∀n ∈ N
∗
. Gọi r, t lần lượt là các số tự nhiên
sao cho M = f (r + 1) − f (r) lớn nhất và m = f (t + 1) − f (t) nhỏ nhất. Ta có
M = f (f (f (r)) + 1) − f (f (f (r))) = f (ar + b + 1) − f (ar + b) = c − b, m
= f (f (f (t)) + 1) − f (f (f (t))) = f (at + b + 1) − f (tr + b) = c − b.
Vậy M = m. Suy ra f (n + 1) − f (n) = c − b, ∀n ∈ N∗
.
Từ đó ta có điều phải chứng minh.
2.3 Các bài tập áp dụng
Các kết quả của các bài toán phương trình hàm số học có ý nghĩa rất
lớn trong việc giải các bài toán phương trình hàm trên tập các số tự nhiên.
Bằng các phép biến đổi đưa về các phương trình hàm đã xét ở trên. Sau
đây ta đi xét một số bài tập áp dụng các kết quả kể trên.
Bài toán 2.11. Cho hàm số f : N
∗
→ N
∗
, thỏa mãn điều kiện
f (mf (n)) = n2
f (m), ∀m, n ∈ N∗
.
Xác định giá trị nhỏ nhất có thể có của f (2010).
Lời giải. Cho m = n = 1 ta có f (f (1)) = f (1), suy ra f (1) = 1.
Cho m = 1 ta có f (f (n)) = n2
, ∀n ∈ N∗
.
Thay m bởi f (m) ta được
f (f (m)f (n)) = n
2
f (f (m)) = n
2
m
2
= f (f (mn)), ∀m, n ∈ N
∗
.
Do f đơn ánh nên f (mn) = f (m)f (n), ∀m, n ∈ N
∗
.
Ta chỉ cần xác định giá trị hàm số tại các số nguyên tố.
Giả sử p là số nguyên tố và f (p) = ab, (a > 1, b > 1).
Viết thuê đề tài giá rẻ trọn gói - KB Zalo/Tele : 0973.287.149
Luanvanmaster.com – Cần Kham Thảo - Kết bạn Zalo/Tele : 0973.287.149
Khi đó f (ab) = f (f (p)) hay f (a)f (b) = p
2
.
Viết thuê đề tài giá rẻ trọn gói - KB Zalo/Tele : 0973.287.149
Luanvanmaster.com – Cần Kham Thảo - Kết bạn Zalo/Tele : 0973.287.149
28
Do p là nguyên tố và f (a) > 1, f (b) > 1 nên chỉ xảy ra khả năng f (a) = f (b)
= p, suy ra a = b.
Vậy f (p) = a
2
và f (a) = p.
Ta sẽ chứng minh a là một số nguyên tố. Thật vậy nếu a = mn, m, n > 1 thì
p = f (a) = f (m)f (n), vô lí vì f (m) > 1 và f (n) > 1.
Tóm lại, nếu p là số nguyên tố thì f (p) hoặc là số nguyên tố, hoặc là bình
phương của một số nguyên tố.
Chú ý rằng không thể xảy ra trường hợp f (p) hoặc f (p) = p
2
với p là số nguyên
tố. Từ đó có thể xây dựng hàm số thỏa mãn yêu cầu bài toán như sau
- Chia tập hợp số nguyên tố thành vô hạn các cặp (p; q) rời
nhau và đặt f (p) = q, f (q) = p2
.
- Với mọi số n = p
α
11 p
α
22 ...p
α
kk ta đặt f (n) = f (p1)
α
1 f (p2)
α
2 ...f (pk)
α
k .
Ta có 2010 = 2
2
.3.5.67 nên giá trị nhỏ nhất có thể có của f (2010) ứng với
hàm số f xác định bởi f (2) = 3, f (3) = 4, f (5) = 7, f (67) = 11 (các giá trị
khác tùy ý miễn thỏa mãn cách xây dựng trên).
Khi đó f (2010) = 3
2
.4.7.11 = 2772.
Bài toán 2.12. Tìm tất cả các hàm số f : N → N, thỏa mãn điều kiện
f (m + f (n)) = f (m) + n, ∀m, n ∈ N.
Lời giải. Dễ thấy f là một đơn ánh.
Cho m = n = 0 được f (f (0)) = f (0) ⇒ f (0) = 0.
Cho m = 0 ta được f (f (n)) = n, ∀n ∈ N.
Bây giờ, thay m bởi f (m) ta có
f (f (m) + f (n)) = f (f (m)) + n = m + n = f (f (m + n)), ∀m, n ∈ N.
Do f là đơn ánh nên f (m + n) = f (m) + f (n), ∀m, n ∈ N.
Suy ra f có dạng f (n) = kn, ∀k, n ∈ N.
Viết thuê đề tài giá rẻ trọn gói - KB Zalo/Tele : 0973.287.149
Luanvanmaster.com – Cần Kham Thảo - Kết bạn Zalo/Tele : 0973.287.149
Thay biểu thức của hàm số vào biểu thức ban đầu, ta tính được k = 1.
Viết thuê đề tài giá rẻ trọn gói - KB Zalo/Tele : 0973.287.149
Luanvanmaster.com – Cần Kham Thảo - Kết bạn Zalo/Tele : 0973.287.149
29
Từ đó suy ra f (n) = n, ∀n ∈ N.
Thử lại thấy hàm này thỏa mãn.
Vậy f (n) = n, ∀n ∈ N là hàm số cần tìm.
Bài toán 2.13 (Pan African 2010). Tồn tại hay không hàm số f : N → Z thỏa
mãn điều kiện
f (x + f (y)) = f (x) − y, ∀x, y ∈ N. (2.12)
Lời giải. Giả sử tồn tại hàm f thỏa mãn (2.12). Ta chứng minh f là song
ánh. Giả sử f (x1) = f (x2). Khi đó:
do (2.12) do (2.12)
f (x) − x1
=
f (x + f (x1)) = f (x + f (x2)
=
f (x) − x2 ⇒ x1 = x2.
Vậy f là đơn ánh. Giả sử t ∈ N, khi đó tồn tại k = 1 + f (f (1) − t) ∈ N và
do (2.12) do (2.12)
f (k)
=
f (1 + f (f (1) − t))
=
f (1)− [f (1) − t] = t.
Vậy f là toàn ánh, suy ra f là song ánh. Từ (2.12) cho x = 1 và y = 0, ta
được f (1 + f (0)) = f (1).
Mà f là song ánh nên từ đây suy ra: 1 + f (0) = 1 hay f (0) = 0. Từ (2.12) cho x = 0,
ta được: f (f (y)) = −y, ∀y ∈ N. Như vậy (2.12) trở thành
f (x + f (y)) = f (x) + f (f (y)), ∀x, y ∈ N. (2.13)
Mà f là song ánh nên từ (2.13) ta suy ra
f (x + y) = f (x) + f (y), ∀x, y ∈ N. (2.14)
Từ (2.14), áp dụng kết quả bài toán 2.1, suy ra
f (x) = ax, ∀x ∈ N, (a = f (1) là hằng số nguyên ).
Thay vào (2.12) ta được:
a(x + ay) = ax − y, ∀x, y ∈ N
⇔a
2
y = −y
Viết thuê đề tài giá rẻ trọn gói - KB Zalo/Tele : 0973.287.149
Luanvanmaster.com – Cần Kham Thảo - Kết bạn Zalo/Tele : 0973.287.149
⇔a
2
= −1.
Điều này vô lí chứng tỏ không tồn tại hàm f thỏa mãn bài ra.
Viết thuê đề tài giá rẻ trọn gói - KB Zalo/Tele : 0973.287.149
Luanvanmaster.com – Cần Kham Thảo - Kết bạn Zalo/Tele : 0973.287.149
30
Bài toán 2.14 (France Selection Test 2007). Tìm tất cả các hàm số f : N → Z thỏa
mãn điều kiện
f (x − y + f (y)) = f (x) + f (y), ∀x, y ∈ N. (2.15)
Lời giải. Kí hiệu P (u, v) chỉ phép thay x bởi u và y bởi v vào (2.15). Đặt f
(0) = a. Khi đó ta có
P (0, 0) ⇒ f (a) = 2a ⇒ f (a) − a = a,
P (0, a) ⇒ f (a) = a + f (a) ⇒ a = 0 ⇒ f (0) = 0,
và
P (0, x) ⇒ f (f (x) − x) = f (x), ∀x ∈ N. (2.16)
Thực hiện P (x, f (y) − y) ta được
f (x − f (y) + y + f (f (y) − y)) = f (x) + f (f (y) − y), ∀x, y ∈ N. (2.17)
Thay (2.16) vào (2.17) ta được
f (x + y) = f (x) + f (y), ∀x, y ∈ N.
Từ đây áp dụng kết quả bài toán 2.1, ta suy ra
f (x) = ax, ∀x ∈ N (a = f (1) là hằng số nguyên ).
Thay vào (2.15) ta được
a(x − y + ay) = ax + ay, ∀x, y ∈ N
⇔(a
2
− 2a)y = 0, ∀y ∈ N.
Vậy a ∈ {0, 2}, do đó có hai hàm số thỏa mãn các yêu cầu đề
bài là f (x) = 0, và f (x) = 2x, ∀x ∈ N.
Bài toán 2.15 (Hong Kong Team Selection Tests 2008). Giả sử hàm số f :
N → Z thỏa mãn f (1) = 1, f (2) = 20, f (−4) = −4 và
Viết thuê đề tài giá rẻ trọn gói - KB Zalo/Tele : 0973.287.149
Luanvanmaster.com – Cần Kham Thảo - Kết bạn Zalo/Tele : 0973.287.149
f (x + y) = f (x) + f (y) + axy(x + y) + bxy + c(x + y) + 4, ∀x, y ∈ N. (2.18)
trong đó a, b, c là các hằng số nguyên.
Viết thuê đề tài giá rẻ trọn gói - KB Zalo/Tele : 0973.287.149
Luanvanmaster.com – Cần Kham Thảo - Kết bạn Zalo/Tele : 0973.287.149
31
a)Tìm hàm số f.
b) Giả sử f (x) ≥ mx2
+ (5m + 1)x + 4m, ∀x ∈ N. Hãy tìm giá trị nhỏ nhất của m.
Lời giải. Kí hiệu P (u, v) chỉ việc thay x bởi u và thay y bởi v vào
(2.18) a) Ta có
P (0, 0) ⇒ f (0) = −4
P (0, y) ⇒ cy = 0, ∀y ∈ N ⇒ c = 0
P (1, −1) ⇒ −4 = 1 + f (−1) − b − 4 ⇒ f (−1) = b − 9
P (−1, −1) ⇒ f (−2) = 2f (−1) − 2a + b + 4 = −2a + 3b − 14
P (−2, −2) ⇒ f (−4) = 2f (−2) − 16a + 4b + 4 = −20a + 10b − 24
Do đó
− 4 = −20a + 10b − 24 ⇔20a − 10b = −20 ⇔2a − b = −2. (2.19)
Và
P (1, 1) ⇒ f (2) = 2f (1) + 2a + b + 4 ⇒ 2a + b = 14. (2.20)
Từ (2.19) và (2.20) suy ra a = 3, b = 8. Vậy
f (x + y) = f (x) + f (y) + 3xy(x + y) + 8xy + 4, ∀x, y ∈ N. (2.21)
Xét hàm số g : N → Z như sau
g(x) = f (x) − (x
3
+ 4x
2
− 4) ⇔f (x) = g(x) + (x
3
+ 4x
2
− 4).
Thay vào (2.21), suy ra, với mọi x, y ∈ N, ta có
g(x + y) + x
3
+ y
3
+ 3xy(x + y) + 4(x + y)
2
− 4 =
= g(x) + g(y) + (x
3
+ 4x
2
− 4) + (y
3
+ 4y
2
− 4) + 3xy(x + y) + 8xy + 4
= g(x) + g(y) + x
3
+ y
3
+ 3xy(x + y) + 4(x + y)
2
− 4.
Như vậy g(x + y) = g(x) + g(y), ∀x, y ∈ N. Từ đây áp dụng kết quả bài toán
2.1, suy ra
Viết thuê đề tài giá rẻ trọn gói - KB Zalo/Tele : 0973.287.149
Luanvanmaster.com – Cần Kham Thảo - Kết bạn Zalo/Tele : 0973.287.149
g(x) = ax, ∀x ∈ Z (a = g(1) là hằng số nguyên ).
Viết thuê đề tài giá rẻ trọn gói - KB Zalo/Tele : 0973.287.149
Luanvanmaster.com – Cần Kham Thảo - Kết bạn Zalo/Tele : 0973.287.149
32
Mà g(1) = f (1) − 1 = 0 nên a = 0. Do đó g(x) = 0, ∀x ∈ N hay
f (x) = x
3
+ 4x
2
− 4, ∀x ∈ N.
Thử lại thấy hàm số trên thỏa mãn yêu cầu bài ra.
b)f (x) ≥ mx
2
+ (5m + 1)x + 4m, ∀x ∈ N. Khi đó
x
3
+ 4x
2
− 4 ≥ mx
2
+ (5m + 1)x + 4m, ∀x ∈ N
⇔x
3
+ 4x
2
− x − 4 ≥ m(x
2
+ 5x + 4), ∀x ∈ N
⇔(x + 1)(x − 1)(x + 4) ≥ m(x + 1)(x + 4), ∀x ∈ N
⇔x − 1 ≥ m, ∀x ∈ N
Vậy giá trị nhỏ nhất của m là −1, đạt được khi x = 0.
Viết thuê đề tài giá rẻ trọn gói - KB Zalo/Tele : 0973.287.149
Luanvanmaster.com – Cần Kham Thảo - Kết bạn Zalo/Tele : 0973.287.149
33
Chương 3
Các dạng toán liên quan

More Related Content

Similar to Luận văn thạc sĩ - Một số lớp phương trình hàm trong số học.doc

Luận văn thạc sĩ toán học
Luận văn thạc sĩ toán họcLuận văn thạc sĩ toán học
Luận văn thạc sĩ toán họcDang Van Ly
 
Vmo 2015-solution-1421633776
Vmo 2015-solution-1421633776Vmo 2015-solution-1421633776
Vmo 2015-solution-1421633776Nguyen Van Tai
 

Similar to Luận văn thạc sĩ - Một số lớp phương trình hàm trong số học.doc (20)

M T So Dạng Toán Ve Dãy So Sinh B I Các Hàm So Sơ Cap.docx
M T So Dạng Toán Ve Dãy So Sinh B I Các Hàm So Sơ Cap.docxM T So Dạng Toán Ve Dãy So Sinh B I Các Hàm So Sơ Cap.docx
M T So Dạng Toán Ve Dãy So Sinh B I Các Hàm So Sơ Cap.docx
 
M T So L P Phương Trình Diophantine.docx
M T So L P Phương Trình Diophantine.docxM T So L P Phương Trình Diophantine.docx
M T So L P Phương Trình Diophantine.docx
 
Bat Đang Thức V I Hàm Loi B Ph N Và Ứng Dụng.docx
Bat Đang Thức V I Hàm Loi B Ph N Và Ứng Dụng.docxBat Đang Thức V I Hàm Loi B Ph N Và Ứng Dụng.docx
Bat Đang Thức V I Hàm Loi B Ph N Và Ứng Dụng.docx
 
Một số thuật toán phân tích số nguyên hiện đại và ứng dụng.doc
Một số thuật toán phân tích số nguyên hiện đại và ứng dụng.docMột số thuật toán phân tích số nguyên hiện đại và ứng dụng.doc
Một số thuật toán phân tích số nguyên hiện đại và ứng dụng.doc
 
Một số lớp đa thức hoán vị trên trường hữu hạn đặc số chẵn.doc
Một số lớp đa thức hoán vị trên trường hữu hạn đặc số chẵn.docMột số lớp đa thức hoán vị trên trường hữu hạn đặc số chẵn.doc
Một số lớp đa thức hoán vị trên trường hữu hạn đặc số chẵn.doc
 
Một Số Vấn Đề Chọn Lọc Về Dãy Số.doc
Một Số Vấn Đề Chọn Lọc Về Dãy Số.docMột Số Vấn Đề Chọn Lọc Về Dãy Số.doc
Một Số Vấn Đề Chọn Lọc Về Dãy Số.doc
 
Bat Phương Trình Hàm Sinh B I Các Đại Lư Ng Trung Bình B C Tùy Ý Và Các Dạng ...
Bat Phương Trình Hàm Sinh B I Các Đại Lư Ng Trung Bình B C Tùy Ý Và Các Dạng ...Bat Phương Trình Hàm Sinh B I Các Đại Lư Ng Trung Bình B C Tùy Ý Và Các Dạng ...
Bat Phương Trình Hàm Sinh B I Các Đại Lư Ng Trung Bình B C Tùy Ý Và Các Dạng ...
 
Số Phức Và Ứng Dụng Vào Giải Toán Phổ Thông Trung Học.doc
Số Phức Và Ứng Dụng Vào Giải Toán Phổ Thông Trung Học.docSố Phức Và Ứng Dụng Vào Giải Toán Phổ Thông Trung Học.doc
Số Phức Và Ứng Dụng Vào Giải Toán Phổ Thông Trung Học.doc
 
Ve H Phương Trình Phi Tuyen Và Ứng Dụng.docx
Ve H Phương Trình Phi Tuyen Và Ứng Dụng.docxVe H Phương Trình Phi Tuyen Và Ứng Dụng.docx
Ve H Phương Trình Phi Tuyen Và Ứng Dụng.docx
 
Xap xỉ diophantine và phân so liên tục trong giải Phương trình pell.docx
Xap xỉ diophantine và phân so liên tục  trong giải Phương trình pell.docxXap xỉ diophantine và phân so liên tục  trong giải Phương trình pell.docx
Xap xỉ diophantine và phân so liên tục trong giải Phương trình pell.docx
 
Luận văn: Giải gần đúng phương trình phi tuyến và phương trình vi phân trên m...
Luận văn: Giải gần đúng phương trình phi tuyến và phương trình vi phân trên m...Luận văn: Giải gần đúng phương trình phi tuyến và phương trình vi phân trên m...
Luận văn: Giải gần đúng phương trình phi tuyến và phương trình vi phân trên m...
 
Luận văn thạc sĩ toán học
Luận văn thạc sĩ toán họcLuận văn thạc sĩ toán học
Luận văn thạc sĩ toán học
 
Xấp xỉ hàm đa điều hòa dưới Bởi hàm green đa cực.doc
Xấp xỉ hàm đa điều hòa dưới Bởi hàm green đa cực.docXấp xỉ hàm đa điều hòa dưới Bởi hàm green đa cực.doc
Xấp xỉ hàm đa điều hòa dưới Bởi hàm green đa cực.doc
 
Bài Toán Phân Hoạch So Nguyên Dương.docx
Bài Toán Phân Hoạch    So Nguyên Dương.docxBài Toán Phân Hoạch    So Nguyên Dương.docx
Bài Toán Phân Hoạch So Nguyên Dương.docx
 
Ứng Dụng Hình Học Tính Toán Để Xác Định Một Miền Chứa Điểm Cho Trƣớc.doc
Ứng Dụng Hình Học Tính Toán Để Xác Định Một Miền Chứa Điểm Cho Trƣớc.docỨng Dụng Hình Học Tính Toán Để Xác Định Một Miền Chứa Điểm Cho Trƣớc.doc
Ứng Dụng Hình Học Tính Toán Để Xác Định Một Miền Chứa Điểm Cho Trƣớc.doc
 
Vmo 2015-solution-1421633776
Vmo 2015-solution-1421633776Vmo 2015-solution-1421633776
Vmo 2015-solution-1421633776
 
M t so dạng toán Liên quan đen xác suat r i rạc và ứng dụng.docx
M t so dạng toán Liên quan đen xác suat r i rạc  và ứng dụng.docxM t so dạng toán Liên quan đen xác suat r i rạc  và ứng dụng.docx
M t so dạng toán Liên quan đen xác suat r i rạc và ứng dụng.docx
 
M T So Dạng Toán Liên Quan Đen Xác Suat R I Rạc Và Ứng Dụng.docx
M T So Dạng Toán Liên Quan Đen Xác Suat R I Rạc Và Ứng Dụng.docxM T So Dạng Toán Liên Quan Đen Xác Suat R I Rạc Và Ứng Dụng.docx
M T So Dạng Toán Liên Quan Đen Xác Suat R I Rạc Và Ứng Dụng.docx
 
Hàm Đơn Đi U, Tựa Đơn Đi U Và M T So Ứng Dụng Của Phép Đơn Đi U Hóa Hàm So.docx
Hàm Đơn Đi U, Tựa Đơn Đi U Và M T So Ứng Dụng Của Phép Đơn Đi U Hóa Hàm So.docxHàm Đơn Đi U, Tựa Đơn Đi U Và M T So Ứng Dụng Của Phép Đơn Đi U Hóa Hàm So.docx
Hàm Đơn Đi U, Tựa Đơn Đi U Và M T So Ứng Dụng Của Phép Đơn Đi U Hóa Hàm So.docx
 
Phương Trình Bậc Bốn Và Các Hệ Thức Hình Học Trong Tứ Giác Hai Tâm.docx
Phương Trình Bậc Bốn Và Các Hệ Thức Hình Học Trong Tứ Giác Hai Tâm.docxPhương Trình Bậc Bốn Và Các Hệ Thức Hình Học Trong Tứ Giác Hai Tâm.docx
Phương Trình Bậc Bốn Và Các Hệ Thức Hình Học Trong Tứ Giác Hai Tâm.docx
 

More from Dịch vụ viết thuê đề tài trọn gói ☎☎☎ Liên hệ ZALO/TELE: 0973.287.149 👍👍

More from Dịch vụ viết thuê đề tài trọn gói ☎☎☎ Liên hệ ZALO/TELE: 0973.287.149 👍👍 (20)

Phân tích các yếu tố ảnh hưởng đến lòng trung thành của nhân viên tại khách s...
Phân tích các yếu tố ảnh hưởng đến lòng trung thành của nhân viên tại khách s...Phân tích các yếu tố ảnh hưởng đến lòng trung thành của nhân viên tại khách s...
Phân tích các yếu tố ảnh hưởng đến lòng trung thành của nhân viên tại khách s...
 
Nghiên cứu về phát triển hệ thống kênh phân phối sản phẩm của các doanh nghiệ...
Nghiên cứu về phát triển hệ thống kênh phân phối sản phẩm của các doanh nghiệ...Nghiên cứu về phát triển hệ thống kênh phân phối sản phẩm của các doanh nghiệ...
Nghiên cứu về phát triển hệ thống kênh phân phối sản phẩm của các doanh nghiệ...
 
CƠ SỞ LÝ LUẬN VỀ THƯƠNG HIỆU.docx
CƠ SỞ LÝ LUẬN VỀ THƯƠNG HIỆU.docxCƠ SỞ LÝ LUẬN VỀ THƯƠNG HIỆU.docx
CƠ SỞ LÝ LUẬN VỀ THƯƠNG HIỆU.docx
 
Cơ sở lý luận của việc nâng cao chất lượng phục vụ tại bộ phận đón tiếp của k...
Cơ sở lý luận của việc nâng cao chất lượng phục vụ tại bộ phận đón tiếp của k...Cơ sở lý luận của việc nâng cao chất lượng phục vụ tại bộ phận đón tiếp của k...
Cơ sở lý luận của việc nâng cao chất lượng phục vụ tại bộ phận đón tiếp của k...
 
Cơ sở lý luận về phát triển thị trƣờng khách inbound dưới góc độ marketing củ...
Cơ sở lý luận về phát triển thị trƣờng khách inbound dưới góc độ marketing củ...Cơ sở lý luận về phát triển thị trƣờng khách inbound dưới góc độ marketing củ...
Cơ sở lý luận về phát triển thị trƣờng khách inbound dưới góc độ marketing củ...
 
Cơ sở lý luận về thị trường và sử dụng marketing nhằm mở rộng thị trường của ...
Cơ sở lý luận về thị trường và sử dụng marketing nhằm mở rộng thị trường của ...Cơ sở lý luận về thị trường và sử dụng marketing nhằm mở rộng thị trường của ...
Cơ sở lý luận về thị trường và sử dụng marketing nhằm mở rộng thị trường của ...
 
Tính toán thiết kế chế tạo và vận hành thử nghiệm hệ thống cấp đông I-Q-F thẳ...
Tính toán thiết kế chế tạo và vận hành thử nghiệm hệ thống cấp đông I-Q-F thẳ...Tính toán thiết kế chế tạo và vận hành thử nghiệm hệ thống cấp đông I-Q-F thẳ...
Tính toán thiết kế chế tạo và vận hành thử nghiệm hệ thống cấp đông I-Q-F thẳ...
 
Tính toán, thiết kế máy sấy bơm nhiệt sấy thanh long cắt lát với năng suất 20...
Tính toán, thiết kế máy sấy bơm nhiệt sấy thanh long cắt lát với năng suất 20...Tính toán, thiết kế máy sấy bơm nhiệt sấy thanh long cắt lát với năng suất 20...
Tính toán, thiết kế máy sấy bơm nhiệt sấy thanh long cắt lát với năng suất 20...
 
Nghiên cứu nhiệt phân gỗ nhằm nâng cao chất lượng sản phẩm than hoa.doc
Nghiên cứu nhiệt phân gỗ nhằm nâng cao chất lượng sản phẩm than hoa.docNghiên cứu nhiệt phân gỗ nhằm nâng cao chất lượng sản phẩm than hoa.doc
Nghiên cứu nhiệt phân gỗ nhằm nâng cao chất lượng sản phẩm than hoa.doc
 
Hoàn thiện quy trình sản xuất thanh long sấy bằng phương pháp sấy đối ...
Hoàn thiện quy trình sản xuất thanh long sấy bằng phương pháp sấy đối ...Hoàn thiện quy trình sản xuất thanh long sấy bằng phương pháp sấy đối ...
Hoàn thiện quy trình sản xuất thanh long sấy bằng phương pháp sấy đối ...
 
Nghiên cứu ứng dụng hệ điều khiển dự báo để điều khiển mức nước bao hơi của n...
Nghiên cứu ứng dụng hệ điều khiển dự báo để điều khiển mức nước bao hơi của n...Nghiên cứu ứng dụng hệ điều khiển dự báo để điều khiển mức nước bao hơi của n...
Nghiên cứu ứng dụng hệ điều khiển dự báo để điều khiển mức nước bao hơi của n...
 
ĐỒ ÁN - BÁO CÁO MÔ HÌNH KHO LẠNH DÀN TRẢI.doc
ĐỒ ÁN - BÁO CÁO MÔ HÌNH KHO LẠNH DÀN TRẢI.docĐỒ ÁN - BÁO CÁO MÔ HÌNH KHO LẠNH DÀN TRẢI.doc
ĐỒ ÁN - BÁO CÁO MÔ HÌNH KHO LẠNH DÀN TRẢI.doc
 
ĐỒ ÁN - Tính toán thiết kế máy sấy khoai lang năng suất 100 kg mẻ.doc
ĐỒ ÁN - Tính toán thiết kế máy sấy khoai lang năng suất 100 kg mẻ.docĐỒ ÁN - Tính toán thiết kế máy sấy khoai lang năng suất 100 kg mẻ.doc
ĐỒ ÁN - Tính toán thiết kế máy sấy khoai lang năng suất 100 kg mẻ.doc
 
Đồ án tốt nghiệp - Sấy bã mía, 9 điểm.doc
Đồ án tốt nghiệp - Sấy bã mía, 9 điểm.docĐồ án tốt nghiệp - Sấy bã mía, 9 điểm.doc
Đồ án tốt nghiệp - Sấy bã mía, 9 điểm.doc
 
Hoàn thiện quy trình sản xuất thanh long sấy bằng phương pháp sấy đối lưu.doc
Hoàn thiện quy trình sản xuất thanh long sấy bằng phương pháp sấy đối lưu.docHoàn thiện quy trình sản xuất thanh long sấy bằng phương pháp sấy đối lưu.doc
Hoàn thiện quy trình sản xuất thanh long sấy bằng phương pháp sấy đối lưu.doc
 
ĐỒ ÁN - Điều khiển lưu lượng không khí trong phòng sạch thông qua biến tần.doc
ĐỒ ÁN - Điều khiển lưu lượng không khí trong phòng sạch thông qua biến tần.docĐỒ ÁN - Điều khiển lưu lượng không khí trong phòng sạch thông qua biến tần.doc
ĐỒ ÁN - Điều khiển lưu lượng không khí trong phòng sạch thông qua biến tần.doc
 
ĐỒ ÁN - Tính toán thiết bị sấy nấm kểu sấy hầm, năng suất nhập liệu 650kgmẻ.doc
ĐỒ ÁN - Tính toán thiết bị sấy nấm kểu sấy hầm, năng suất nhập liệu 650kgmẻ.docĐỒ ÁN - Tính toán thiết bị sấy nấm kểu sấy hầm, năng suất nhập liệu 650kgmẻ.doc
ĐỒ ÁN - Tính toán thiết bị sấy nấm kểu sấy hầm, năng suất nhập liệu 650kgmẻ.doc
 
Thiết kế nhà máy sản xuất bia năng suất 91,8 triệu lít sản phẩm năm.docx
Thiết kế nhà máy sản xuất bia năng suất 91,8 triệu lít sản phẩm năm.docxThiết kế nhà máy sản xuất bia năng suất 91,8 triệu lít sản phẩm năm.docx
Thiết kế nhà máy sản xuất bia năng suất 91,8 triệu lít sản phẩm năm.docx
 
Tính toán thiết kế hệ thống sấy thùng quay sấy cà phê nhân theo năng suất nhậ...
Tính toán thiết kế hệ thống sấy thùng quay sấy cà phê nhân theo năng suất nhậ...Tính toán thiết kế hệ thống sấy thùng quay sấy cà phê nhân theo năng suất nhậ...
Tính toán thiết kế hệ thống sấy thùng quay sấy cà phê nhân theo năng suất nhậ...
 
Thiết kế hệ thống sấy thùng quay sấy bắp với năng suất 800 kgh.docx
Thiết kế hệ thống sấy thùng quay sấy bắp với năng suất 800 kgh.docxThiết kế hệ thống sấy thùng quay sấy bắp với năng suất 800 kgh.docx
Thiết kế hệ thống sấy thùng quay sấy bắp với năng suất 800 kgh.docx
 

Recently uploaded

Bài tập nhóm Kỹ Năng Gỉai Quyết Tranh Chấp Lao Động (1).pptx
Bài tập nhóm Kỹ Năng Gỉai Quyết Tranh Chấp Lao Động (1).pptxBài tập nhóm Kỹ Năng Gỉai Quyết Tranh Chấp Lao Động (1).pptx
Bài tập nhóm Kỹ Năng Gỉai Quyết Tranh Chấp Lao Động (1).pptxDungxPeach
 
3-BẢNG MÃ LỖI CỦA CÁC HÃNG ĐIỀU HÒA .pdf - ĐIỆN LẠNH BÁCH KHOA HÀ NỘI
3-BẢNG MÃ LỖI CỦA CÁC HÃNG ĐIỀU HÒA .pdf - ĐIỆN LẠNH BÁCH KHOA HÀ NỘI3-BẢNG MÃ LỖI CỦA CÁC HÃNG ĐIỀU HÒA .pdf - ĐIỆN LẠNH BÁCH KHOA HÀ NỘI
3-BẢNG MÃ LỖI CỦA CÁC HÃNG ĐIỀU HÒA .pdf - ĐIỆN LẠNH BÁCH KHOA HÀ NỘIĐiện Lạnh Bách Khoa Hà Nội
 
Danh sách sinh viên tốt nghiệp Đại học - Cao đẳng Trường Đại học Phú Yên năm ...
Danh sách sinh viên tốt nghiệp Đại học - Cao đẳng Trường Đại học Phú Yên năm ...Danh sách sinh viên tốt nghiệp Đại học - Cao đẳng Trường Đại học Phú Yên năm ...
Danh sách sinh viên tốt nghiệp Đại học - Cao đẳng Trường Đại học Phú Yên năm ...hoangtuansinh1
 
powerpoint mẫu họp phụ huynh cuối kì 2 học sinh lớp 7 bgs
powerpoint mẫu họp phụ huynh cuối kì 2 học sinh lớp 7 bgspowerpoint mẫu họp phụ huynh cuối kì 2 học sinh lớp 7 bgs
powerpoint mẫu họp phụ huynh cuối kì 2 học sinh lớp 7 bgsNmmeomeo
 
GIÁO ÁN DẠY THÊM (KẾ HOẠCH BÀI DẠY BUỔI 2) - TIẾNG ANH 7 GLOBAL SUCCESS (2 CỘ...
GIÁO ÁN DẠY THÊM (KẾ HOẠCH BÀI DẠY BUỔI 2) - TIẾNG ANH 7 GLOBAL SUCCESS (2 CỘ...GIÁO ÁN DẠY THÊM (KẾ HOẠCH BÀI DẠY BUỔI 2) - TIẾNG ANH 7 GLOBAL SUCCESS (2 CỘ...
GIÁO ÁN DẠY THÊM (KẾ HOẠCH BÀI DẠY BUỔI 2) - TIẾNG ANH 7 GLOBAL SUCCESS (2 CỘ...Nguyen Thanh Tu Collection
 
30 ĐỀ PHÁT TRIỂN THEO CẤU TRÚC ĐỀ MINH HỌA BGD NGÀY 22-3-2024 KỲ THI TỐT NGHI...
30 ĐỀ PHÁT TRIỂN THEO CẤU TRÚC ĐỀ MINH HỌA BGD NGÀY 22-3-2024 KỲ THI TỐT NGHI...30 ĐỀ PHÁT TRIỂN THEO CẤU TRÚC ĐỀ MINH HỌA BGD NGÀY 22-3-2024 KỲ THI TỐT NGHI...
30 ĐỀ PHÁT TRIỂN THEO CẤU TRÚC ĐỀ MINH HỌA BGD NGÀY 22-3-2024 KỲ THI TỐT NGHI...Nguyen Thanh Tu Collection
 
BỘ LUYỆN NGHE VÀO 10 TIẾNG ANH DẠNG TRẮC NGHIỆM 4 CÂU TRẢ LỜI - CÓ FILE NGHE.pdf
BỘ LUYỆN NGHE VÀO 10 TIẾNG ANH DẠNG TRẮC NGHIỆM 4 CÂU TRẢ LỜI - CÓ FILE NGHE.pdfBỘ LUYỆN NGHE VÀO 10 TIẾNG ANH DẠNG TRẮC NGHIỆM 4 CÂU TRẢ LỜI - CÓ FILE NGHE.pdf
BỘ LUYỆN NGHE VÀO 10 TIẾNG ANH DẠNG TRẮC NGHIỆM 4 CÂU TRẢ LỜI - CÓ FILE NGHE.pdfNguyen Thanh Tu Collection
 
Trắc nghiệm CHƯƠNG 5 môn Chủ nghĩa xã hội
Trắc nghiệm CHƯƠNG 5 môn Chủ nghĩa xã hộiTrắc nghiệm CHƯƠNG 5 môn Chủ nghĩa xã hội
Trắc nghiệm CHƯƠNG 5 môn Chủ nghĩa xã hộiNgocNguyen591215
 
1.DOANNGOCPHUONGTHAO-APDUNGSTEMTHIETKEBTHHHGIUPHSHOCHIEUQUA (1).docx
1.DOANNGOCPHUONGTHAO-APDUNGSTEMTHIETKEBTHHHGIUPHSHOCHIEUQUA (1).docx1.DOANNGOCPHUONGTHAO-APDUNGSTEMTHIETKEBTHHHGIUPHSHOCHIEUQUA (1).docx
1.DOANNGOCPHUONGTHAO-APDUNGSTEMTHIETKEBTHHHGIUPHSHOCHIEUQUA (1).docxTHAO316680
 
Access: Chuong III Thiet ke truy van Query.ppt
Access: Chuong III Thiet ke truy van Query.pptAccess: Chuong III Thiet ke truy van Query.ppt
Access: Chuong III Thiet ke truy van Query.pptPhamThiThuThuy1
 
Đề thi tin học HK2 lớp 3 Chân Trời Sáng Tạo
Đề thi tin học HK2 lớp 3 Chân Trời Sáng TạoĐề thi tin học HK2 lớp 3 Chân Trời Sáng Tạo
Đề thi tin học HK2 lớp 3 Chân Trời Sáng Tạowindcances
 
xemsomenh.com-Vòng Thái Tuế và Ý Nghĩa Các Sao Tại Cung Mệnh.pdf
xemsomenh.com-Vòng Thái Tuế và Ý Nghĩa Các Sao Tại Cung Mệnh.pdfxemsomenh.com-Vòng Thái Tuế và Ý Nghĩa Các Sao Tại Cung Mệnh.pdf
xemsomenh.com-Vòng Thái Tuế và Ý Nghĩa Các Sao Tại Cung Mệnh.pdfXem Số Mệnh
 
Giáo trình nhập môn lập trình - Đặng Bình Phương
Giáo trình nhập môn lập trình - Đặng Bình PhươngGiáo trình nhập môn lập trình - Đặng Bình Phương
Giáo trình nhập môn lập trình - Đặng Bình Phươnghazzthuan
 
C6. Van de dan toc va ton giao ....pdf . Chu nghia xa hoi
C6. Van de dan toc va ton giao ....pdf . Chu nghia xa hoiC6. Van de dan toc va ton giao ....pdf . Chu nghia xa hoi
C6. Van de dan toc va ton giao ....pdf . Chu nghia xa hoidnghia2002
 
30 ĐỀ PHÁT TRIỂN THEO CẤU TRÚC ĐỀ MINH HỌA BGD NGÀY 22-3-2024 KỲ THI TỐT NGHI...
30 ĐỀ PHÁT TRIỂN THEO CẤU TRÚC ĐỀ MINH HỌA BGD NGÀY 22-3-2024 KỲ THI TỐT NGHI...30 ĐỀ PHÁT TRIỂN THEO CẤU TRÚC ĐỀ MINH HỌA BGD NGÀY 22-3-2024 KỲ THI TỐT NGHI...
30 ĐỀ PHÁT TRIỂN THEO CẤU TRÚC ĐỀ MINH HỌA BGD NGÀY 22-3-2024 KỲ THI TỐT NGHI...Nguyen Thanh Tu Collection
 
SD-05_Xây dựng website bán váy Lolita Alice - Phùng Thị Thúy Hiền PH 2 7 8 6 ...
SD-05_Xây dựng website bán váy Lolita Alice - Phùng Thị Thúy Hiền PH 2 7 8 6 ...SD-05_Xây dựng website bán váy Lolita Alice - Phùng Thị Thúy Hiền PH 2 7 8 6 ...
SD-05_Xây dựng website bán váy Lolita Alice - Phùng Thị Thúy Hiền PH 2 7 8 6 ...ChuThNgnFEFPLHN
 
ĐỀ CHÍNH THỨC KỲ THI TUYỂN SINH VÀO LỚP 10 THPT CÁC TỈNH THÀNH NĂM HỌC 2020 –...
ĐỀ CHÍNH THỨC KỲ THI TUYỂN SINH VÀO LỚP 10 THPT CÁC TỈNH THÀNH NĂM HỌC 2020 –...ĐỀ CHÍNH THỨC KỲ THI TUYỂN SINH VÀO LỚP 10 THPT CÁC TỈNH THÀNH NĂM HỌC 2020 –...
ĐỀ CHÍNH THỨC KỲ THI TUYỂN SINH VÀO LỚP 10 THPT CÁC TỈNH THÀNH NĂM HỌC 2020 –...Nguyen Thanh Tu Collection
 
TÀI LIỆU BỒI DƯỠNG HỌC SINH GIỎI KỸ NĂNG VIẾT ĐOẠN VĂN NGHỊ LUẬN XÃ HỘI 200 C...
TÀI LIỆU BỒI DƯỠNG HỌC SINH GIỎI KỸ NĂNG VIẾT ĐOẠN VĂN NGHỊ LUẬN XÃ HỘI 200 C...TÀI LIỆU BỒI DƯỠNG HỌC SINH GIỎI KỸ NĂNG VIẾT ĐOẠN VĂN NGHỊ LUẬN XÃ HỘI 200 C...
TÀI LIỆU BỒI DƯỠNG HỌC SINH GIỎI KỸ NĂNG VIẾT ĐOẠN VĂN NGHỊ LUẬN XÃ HỘI 200 C...Nguyen Thanh Tu Collection
 
TUYỂN TẬP ĐỀ THI GIỮA KÌ, CUỐI KÌ 2 MÔN VẬT LÍ LỚP 11 THEO HÌNH THỨC THI MỚI ...
TUYỂN TẬP ĐỀ THI GIỮA KÌ, CUỐI KÌ 2 MÔN VẬT LÍ LỚP 11 THEO HÌNH THỨC THI MỚI ...TUYỂN TẬP ĐỀ THI GIỮA KÌ, CUỐI KÌ 2 MÔN VẬT LÍ LỚP 11 THEO HÌNH THỨC THI MỚI ...
TUYỂN TẬP ĐỀ THI GIỮA KÌ, CUỐI KÌ 2 MÔN VẬT LÍ LỚP 11 THEO HÌNH THỨC THI MỚI ...Nguyen Thanh Tu Collection
 
bài tập lớn môn kiến trúc máy tính và hệ điều hành
bài tập lớn môn kiến trúc máy tính và hệ điều hànhbài tập lớn môn kiến trúc máy tính và hệ điều hành
bài tập lớn môn kiến trúc máy tính và hệ điều hànhdangdinhkien2k4
 

Recently uploaded (20)

Bài tập nhóm Kỹ Năng Gỉai Quyết Tranh Chấp Lao Động (1).pptx
Bài tập nhóm Kỹ Năng Gỉai Quyết Tranh Chấp Lao Động (1).pptxBài tập nhóm Kỹ Năng Gỉai Quyết Tranh Chấp Lao Động (1).pptx
Bài tập nhóm Kỹ Năng Gỉai Quyết Tranh Chấp Lao Động (1).pptx
 
3-BẢNG MÃ LỖI CỦA CÁC HÃNG ĐIỀU HÒA .pdf - ĐIỆN LẠNH BÁCH KHOA HÀ NỘI
3-BẢNG MÃ LỖI CỦA CÁC HÃNG ĐIỀU HÒA .pdf - ĐIỆN LẠNH BÁCH KHOA HÀ NỘI3-BẢNG MÃ LỖI CỦA CÁC HÃNG ĐIỀU HÒA .pdf - ĐIỆN LẠNH BÁCH KHOA HÀ NỘI
3-BẢNG MÃ LỖI CỦA CÁC HÃNG ĐIỀU HÒA .pdf - ĐIỆN LẠNH BÁCH KHOA HÀ NỘI
 
Danh sách sinh viên tốt nghiệp Đại học - Cao đẳng Trường Đại học Phú Yên năm ...
Danh sách sinh viên tốt nghiệp Đại học - Cao đẳng Trường Đại học Phú Yên năm ...Danh sách sinh viên tốt nghiệp Đại học - Cao đẳng Trường Đại học Phú Yên năm ...
Danh sách sinh viên tốt nghiệp Đại học - Cao đẳng Trường Đại học Phú Yên năm ...
 
powerpoint mẫu họp phụ huynh cuối kì 2 học sinh lớp 7 bgs
powerpoint mẫu họp phụ huynh cuối kì 2 học sinh lớp 7 bgspowerpoint mẫu họp phụ huynh cuối kì 2 học sinh lớp 7 bgs
powerpoint mẫu họp phụ huynh cuối kì 2 học sinh lớp 7 bgs
 
GIÁO ÁN DẠY THÊM (KẾ HOẠCH BÀI DẠY BUỔI 2) - TIẾNG ANH 7 GLOBAL SUCCESS (2 CỘ...
GIÁO ÁN DẠY THÊM (KẾ HOẠCH BÀI DẠY BUỔI 2) - TIẾNG ANH 7 GLOBAL SUCCESS (2 CỘ...GIÁO ÁN DẠY THÊM (KẾ HOẠCH BÀI DẠY BUỔI 2) - TIẾNG ANH 7 GLOBAL SUCCESS (2 CỘ...
GIÁO ÁN DẠY THÊM (KẾ HOẠCH BÀI DẠY BUỔI 2) - TIẾNG ANH 7 GLOBAL SUCCESS (2 CỘ...
 
30 ĐỀ PHÁT TRIỂN THEO CẤU TRÚC ĐỀ MINH HỌA BGD NGÀY 22-3-2024 KỲ THI TỐT NGHI...
30 ĐỀ PHÁT TRIỂN THEO CẤU TRÚC ĐỀ MINH HỌA BGD NGÀY 22-3-2024 KỲ THI TỐT NGHI...30 ĐỀ PHÁT TRIỂN THEO CẤU TRÚC ĐỀ MINH HỌA BGD NGÀY 22-3-2024 KỲ THI TỐT NGHI...
30 ĐỀ PHÁT TRIỂN THEO CẤU TRÚC ĐỀ MINH HỌA BGD NGÀY 22-3-2024 KỲ THI TỐT NGHI...
 
BỘ LUYỆN NGHE VÀO 10 TIẾNG ANH DẠNG TRẮC NGHIỆM 4 CÂU TRẢ LỜI - CÓ FILE NGHE.pdf
BỘ LUYỆN NGHE VÀO 10 TIẾNG ANH DẠNG TRẮC NGHIỆM 4 CÂU TRẢ LỜI - CÓ FILE NGHE.pdfBỘ LUYỆN NGHE VÀO 10 TIẾNG ANH DẠNG TRẮC NGHIỆM 4 CÂU TRẢ LỜI - CÓ FILE NGHE.pdf
BỘ LUYỆN NGHE VÀO 10 TIẾNG ANH DẠNG TRẮC NGHIỆM 4 CÂU TRẢ LỜI - CÓ FILE NGHE.pdf
 
Trắc nghiệm CHƯƠNG 5 môn Chủ nghĩa xã hội
Trắc nghiệm CHƯƠNG 5 môn Chủ nghĩa xã hộiTrắc nghiệm CHƯƠNG 5 môn Chủ nghĩa xã hội
Trắc nghiệm CHƯƠNG 5 môn Chủ nghĩa xã hội
 
1.DOANNGOCPHUONGTHAO-APDUNGSTEMTHIETKEBTHHHGIUPHSHOCHIEUQUA (1).docx
1.DOANNGOCPHUONGTHAO-APDUNGSTEMTHIETKEBTHHHGIUPHSHOCHIEUQUA (1).docx1.DOANNGOCPHUONGTHAO-APDUNGSTEMTHIETKEBTHHHGIUPHSHOCHIEUQUA (1).docx
1.DOANNGOCPHUONGTHAO-APDUNGSTEMTHIETKEBTHHHGIUPHSHOCHIEUQUA (1).docx
 
Access: Chuong III Thiet ke truy van Query.ppt
Access: Chuong III Thiet ke truy van Query.pptAccess: Chuong III Thiet ke truy van Query.ppt
Access: Chuong III Thiet ke truy van Query.ppt
 
Đề thi tin học HK2 lớp 3 Chân Trời Sáng Tạo
Đề thi tin học HK2 lớp 3 Chân Trời Sáng TạoĐề thi tin học HK2 lớp 3 Chân Trời Sáng Tạo
Đề thi tin học HK2 lớp 3 Chân Trời Sáng Tạo
 
xemsomenh.com-Vòng Thái Tuế và Ý Nghĩa Các Sao Tại Cung Mệnh.pdf
xemsomenh.com-Vòng Thái Tuế và Ý Nghĩa Các Sao Tại Cung Mệnh.pdfxemsomenh.com-Vòng Thái Tuế và Ý Nghĩa Các Sao Tại Cung Mệnh.pdf
xemsomenh.com-Vòng Thái Tuế và Ý Nghĩa Các Sao Tại Cung Mệnh.pdf
 
Giáo trình nhập môn lập trình - Đặng Bình Phương
Giáo trình nhập môn lập trình - Đặng Bình PhươngGiáo trình nhập môn lập trình - Đặng Bình Phương
Giáo trình nhập môn lập trình - Đặng Bình Phương
 
C6. Van de dan toc va ton giao ....pdf . Chu nghia xa hoi
C6. Van de dan toc va ton giao ....pdf . Chu nghia xa hoiC6. Van de dan toc va ton giao ....pdf . Chu nghia xa hoi
C6. Van de dan toc va ton giao ....pdf . Chu nghia xa hoi
 
30 ĐỀ PHÁT TRIỂN THEO CẤU TRÚC ĐỀ MINH HỌA BGD NGÀY 22-3-2024 KỲ THI TỐT NGHI...
30 ĐỀ PHÁT TRIỂN THEO CẤU TRÚC ĐỀ MINH HỌA BGD NGÀY 22-3-2024 KỲ THI TỐT NGHI...30 ĐỀ PHÁT TRIỂN THEO CẤU TRÚC ĐỀ MINH HỌA BGD NGÀY 22-3-2024 KỲ THI TỐT NGHI...
30 ĐỀ PHÁT TRIỂN THEO CẤU TRÚC ĐỀ MINH HỌA BGD NGÀY 22-3-2024 KỲ THI TỐT NGHI...
 
SD-05_Xây dựng website bán váy Lolita Alice - Phùng Thị Thúy Hiền PH 2 7 8 6 ...
SD-05_Xây dựng website bán váy Lolita Alice - Phùng Thị Thúy Hiền PH 2 7 8 6 ...SD-05_Xây dựng website bán váy Lolita Alice - Phùng Thị Thúy Hiền PH 2 7 8 6 ...
SD-05_Xây dựng website bán váy Lolita Alice - Phùng Thị Thúy Hiền PH 2 7 8 6 ...
 
ĐỀ CHÍNH THỨC KỲ THI TUYỂN SINH VÀO LỚP 10 THPT CÁC TỈNH THÀNH NĂM HỌC 2020 –...
ĐỀ CHÍNH THỨC KỲ THI TUYỂN SINH VÀO LỚP 10 THPT CÁC TỈNH THÀNH NĂM HỌC 2020 –...ĐỀ CHÍNH THỨC KỲ THI TUYỂN SINH VÀO LỚP 10 THPT CÁC TỈNH THÀNH NĂM HỌC 2020 –...
ĐỀ CHÍNH THỨC KỲ THI TUYỂN SINH VÀO LỚP 10 THPT CÁC TỈNH THÀNH NĂM HỌC 2020 –...
 
TÀI LIỆU BỒI DƯỠNG HỌC SINH GIỎI KỸ NĂNG VIẾT ĐOẠN VĂN NGHỊ LUẬN XÃ HỘI 200 C...
TÀI LIỆU BỒI DƯỠNG HỌC SINH GIỎI KỸ NĂNG VIẾT ĐOẠN VĂN NGHỊ LUẬN XÃ HỘI 200 C...TÀI LIỆU BỒI DƯỠNG HỌC SINH GIỎI KỸ NĂNG VIẾT ĐOẠN VĂN NGHỊ LUẬN XÃ HỘI 200 C...
TÀI LIỆU BỒI DƯỠNG HỌC SINH GIỎI KỸ NĂNG VIẾT ĐOẠN VĂN NGHỊ LUẬN XÃ HỘI 200 C...
 
TUYỂN TẬP ĐỀ THI GIỮA KÌ, CUỐI KÌ 2 MÔN VẬT LÍ LỚP 11 THEO HÌNH THỨC THI MỚI ...
TUYỂN TẬP ĐỀ THI GIỮA KÌ, CUỐI KÌ 2 MÔN VẬT LÍ LỚP 11 THEO HÌNH THỨC THI MỚI ...TUYỂN TẬP ĐỀ THI GIỮA KÌ, CUỐI KÌ 2 MÔN VẬT LÍ LỚP 11 THEO HÌNH THỨC THI MỚI ...
TUYỂN TẬP ĐỀ THI GIỮA KÌ, CUỐI KÌ 2 MÔN VẬT LÍ LỚP 11 THEO HÌNH THỨC THI MỚI ...
 
bài tập lớn môn kiến trúc máy tính và hệ điều hành
bài tập lớn môn kiến trúc máy tính và hệ điều hànhbài tập lớn môn kiến trúc máy tính và hệ điều hành
bài tập lớn môn kiến trúc máy tính và hệ điều hành
 

Luận văn thạc sĩ - Một số lớp phương trình hàm trong số học.doc

  • 1. Viết thuê đề tài giá rẻ trọn gói - KB Zalo/Tele : 0973.287.149 Luanvanmaster.com – Cần Kham Thảo - Kết bạn Zalo/Tele : 0973.287.149 ĐẠI HỌC THÁI NGUYÊN TRƯỜNG ĐẠI HỌC KHOA HỌC PHẠM THANH LINH MỘT SỐ LỚP PHƯƠNG TRÌNH HÀM TRONG SỐ HỌC LUẬN VĂN THẠC SĨ TOÁN HỌC Thái Nguyên -
  • 2. Viết thuê đề tài giá rẻ trọn gói - KB Zalo/Tele : 0973.287.149 Luanvanmaster.com – Cần Kham Thảo - Kết bạn Zalo/Tele : 0973.287.149 Mục lục Lời cảm ơn iii Mở đầu 1 1 Lớp hàm số học cơ bản 3 1.1 Hàm số học . . . . . . . . . . . . . . . . . . . . . . . . . . . . . . 3 1.1.1 Hàm nhân tính . . . . . . . . . . . . . . . . . . . . . . . . 3 1.1.2 Hàm nhân tính mạnh . . . . . . . . . . . . . . . . . . . . . 11 1.2 Hàm số xác định trên tập các số nguyên . . . . . . . . . . . . . . . 11 1.2.1 Hàm cộng tính trên tập các số nguyên . . . . . . . . . . . . 11 1.2.2 Hàm nhân tính trên tập các số nguyên . . . . . . . . . . . . 11 1.2.3 Lớp hàm tuần hoàn, phản tuần hoàn cộng tính, nhân tính . . 12 1.3 Một số bài tập áp dụng . . . . . . . . . . . . . . . . . . . . . . . . 14 2 Các phương trình hàm số học 20 2.1 Hàm chuyển đổi các phép tính số học . . . . . . . . . . . . . . . . 20 2.1.1 Hàm chuyển đổi phép cộng thành phép cộng . . . . . . . . 20 2.1.2 Hàm chuyển đổi phép cộng thành phép nhân . . . . . . . . 21 2.1.3 Hàm chuyển đổi phép nhân thành phép cộng . . . . . . . . 22 2.2 Các dạng toán xác định dãy số liên quan . . . . . . . . . . . . . . . 22 2.3 Các bài tập áp dụng . . . . . . . . . . . . . . . . . . . . . . . . . . 27 3 Các dạng toán liên quan 33 3.1 Phương trình hàm trên N và trên Z . . . . . . . . . . . . . . . . . . 33
  • 3. Viết thuê đề tài giá rẻ trọn gói - KB Zalo/Tele : 0973.287.149 Luanvanmaster.com – Cần Kham Thảo - Kết bạn Zalo/Tele : 0973.287.149 ii 3.1.1 Lớp các bài toán áp dụng nguyên lý quy nạp toán học . . . . 33 3.1.2 Lớp các bài toán áp dụng nguyên lí cực hạn . . . . . . . . . 42 3.1.3 Lớp các bài toán áp dụng hệ đếm cơ số . . . . . . . . . . . 46 3.1.4 Lớp các bài toán áp dụng các tính chất số học . . . . . . . . 53 3.1.5 Lớp các bài toán áp dụng các tính chất dãy số . . . . . . . . 62 3.1.6 Lớp các bài toán áp dụng các tính chất hàm số . . . . . . . . 66 3.2 Phương trình hàm trên Q . . . . . . . . . . . . . . . . . . . . . . . 73 Kết luận và Đề nghị 81 Tài liệu tham khảo 82
  • 4. Viết thuê đề tài giá rẻ trọn gói - KB Zalo/Tele : 0973.287.149 Luanvanmaster.com – Cần Kham Thảo - Kết bạn Zalo/Tele : 0973.287.149 iii Lời cảm ơn Luận văn này được thực hiện và hoàn thành tại Trường Đại học Khoa học - Đại học Thái Nguyên. Đầu tiên em xin bày tỏ lòng biết ơn sâu sắc nhất tới người thầy đáng kính GS.TSKH. Nguyễn Văn Mậu - Trường Đại học Khoa học Tự nhiên - Đại học Quốc gia Hà Nội. Thầy đã dành nhiều thời gian hướng dẫn và giải đáp các thắc mắc trong suốt quá trình xây dựng đề cương, làm và hoàn thiện luận văn. Em xin gửi lời cảm ơn chân thành nhất đến các Thầy cô khoa Toán, phòng Đào tạo sau Đại học, trường Đại học Khoa học - Đại học Thái Nguyên, cùng các Thầy cô giáo tham gia trực tiếp giảng dạy lớp cao học khóa 1/2014 - 1/2016. Đồng thời tôi xin gửi lời cảm ơn tới tập thể lớp K7C Cao học Toán - Đại học Khoa học đã động viên giúp đỡ tôi trong quá trình học tập và làm luận văn này. Em xin chân thành cảm ơn! Thái Nguyên, 2015 Phạm Thanh Linh Học viên Cao học Toán K7C, Trường ĐH Khoa học - ĐH Thái Nguyên
  • 5. Viết thuê đề tài giá rẻ trọn gói - KB Zalo/Tele : 0973.287.149 Luanvanmaster.com – Cần Kham Thảo - Kết bạn Zalo/Tele : 0973.287.149 1 Mở đầu Phương trình hàm là một lĩnh vực quan trọng của giải tích. Bài toán giải phương trình hàm có lẽ là một trong những bài toán lâu đời nhất của giải tích. Nhu cầu giải phương trình hàm xuất hiện ngay khi bắt đầu có lí thuyết hàm số. Nhiều phương trình hàm xuất phát từ nhu cầu thực tế của Toán học hoặc của các ngành khoa học khác. Các nhà toán học đã có công nghiên cứu và đặt nền móng cho phương trình hàm phải kể đến: Nicole Oresme, Gregory of Saint-Vincent, Augusstin-Louis Cauchy, Carl Friedrich Gauss, D’Alembert . . . Ngày nay ở nước ta phương trình hàm được giảng dạy theo chuyên đề ở các trường THPT chuyên. Các dạng toán phương trình hàm trong số học là dạng toán khó thường xuất hiện trong các kì thi học sinh giỏi cấp tỉnh, thành phố, cấp quốc gia, khu vực và quốc tế. Một phương trình hàm bao gồm ba thành phần chính: Tập nguồn, tập đích, phương trình hay hệ phương trình hàm. Từ ba thành phần này có sự phân loại tương ứng. Phương trình hàm trên N, phương trình hàm trên Z, phương trình hàm trên Q, phương trình hàm trên R · · · ; phương trình hàm với một biến tự do, hai biến tự do, nhiều biến tự do, phương trình hàm chuyển đổi các giá trị trung bình · · · ; phương trình hàm trên lớp hàm khả vi, phương trình hàm trên lớp hàm liên tục · · · Việc xác định rõ cấu trúc và tính chất của tập nguồn, tập đích và các điều kiện ràng buộc sẽ quyết định sự thành công hay thất bại khi giải phương trình hàm. Điều này có thể thấy rõ qua phương trình hàm Cauchy. Bài toán tổng quát tìm tất cả các hàm số f : R → R thỏa mãn phương trình f (x + y) = f (x) + f (y) với mọi x, y ∈ R theo một nghĩa nào đó không có lời giải, thế nhưng với giới hạn tập nguồn, tập đích, các tính
  • 6. Viết thuê đề tài giá rẻ trọn gói - KB Zalo/Tele : 0973.287.149 Luanvanmaster.com – Cần Kham Thảo - Kết bạn Zalo/Tele : 0973.287.149 chất của hàm số (đơn điệu, liên tục · · · ) thì phương trình hàm này giải được trọn vẹn. Trong luận văn này tôi chỉ xin trình bày về một lớp phương trình hàm mà
  • 7. Viết thuê đề tài giá rẻ trọn gói - KB Zalo/Tele : 0973.287.149 Luanvanmaster.com – Cần Kham Thảo - Kết bạn Zalo/Tele : 0973.287.149 2 tập nguồn xác định trên N, Z, Q. Trên thực tế khi tìm hiểu lớp các phương trình hàm này xuất hiện rất nhiều trong các kì thi Olympic toán các nước, khu vực và quốc tế. Xuất phát từ thực tế đó, dưới sự định hướng và hướng dẫn nhiệt tình của GS.TSKH. Nguyễn Văn Mậu, tôi đã tiến hành nghiên cứu về đề tài "Một số lớp phương trình hàm trong số học" nhằm mục đích học tập và nghiên cứu sâu về một chuyên đề khó của toán sơ cấp. Cấu trúc luận văn gồm 3 chương. Chương 1. Lớp hàm số học cơ bản Trong chương này sẽ trình bày định nghĩa, các tính chất của các hàm số học cơ bản và một số ứng dụng của chúng vào việc giải các bài toán sơ cấp. Chương 2. Các phương trình hàm số học Trong chương này sẽ trình bày về hàm chuyển đổi các phép tính số học, các dạng toán xác định dãy số liên quan và các bài tập áp dụng. Chương 3. Các dạng toán liên quan Trong chương này sẽ trình bày các dạng toán từ các đề thi Olympic các nước và quốc tế liên quan đến tính toán, ước lượng và các tính chất số học (nguyên tố, chính phương, tính đơn điệu, tính tuần hoàn. . . ) của các hàm số trên các tập số tự nhiên, tập số nguyên và tập số hữu tỷ. Dù đã nghiêm túc nghiên cứu và rất cố gắng thực hiện luận văn, nhưng do năng lực của bản thân cùng nhiều lý do khác, luận văn chắc chắn không tránh khỏi những thiếu sót. Kính mong được sự góp ý của các Thầy cô, các bạn và các anh chị đồng nghiệp để luận văn này hoàn chỉnh và có nhiều ý nghĩa hơn. Thái Nguyên, ngày 28 tháng 11 năm 2015 Phạm Thanh Linh Học viên Cao học Toán lớp C, khóa 01/2014 - 01/2016 Chuyên ngành phương pháp Toán sơ cấp Trường Đại học Khoa học - Đại học Thái Nguyên
  • 8. Viết thuê đề tài giá rẻ trọn gói - KB Zalo/Tele : 0973.287.149 Luanvanmaster.com – Cần Kham Thảo - Kết bạn Zalo/Tele : 0973.287.149 Email: bobaki2010@gmail.com
  • 9. Viết thuê đề tài giá rẻ trọn gói - KB Zalo/Tele : 0973.287.149 Luanvanmaster.com – Cần Kham Thảo - Kết bạn Zalo/Tele : 0973.287.149 3 Chương 1 Lớp hàm số học cơ bản Trong chương trình phổ thông, các bài toán số học đóng vai trò quan trọng trong việc hình thành tư duy toán học. Việc tìm hiểu và sử dụng các hàm số học đã giải quyết được những lớp bài toán cơ bản trong các bài toán sơ cấp và lớp các bài toán phương trình hàm trong số học. Trong chương này sẽ trình bày định nghĩa, các tính chất của các hàm số học cơ bản và một số ứng dụng của chúng vào việc giải các bài toán sơ cấp. 1.1 Hàm số học Định nghĩa 1.1 ([9]). Hàm số học tức là hàm xác định trên tập các số nguyên dương. 1.1.1 Hàm nhân tính Định nghĩa 1.2 ([9]). Một hàm số học f được gọi là nhân tính nếu với mọi m, n nguyên tố cùng nhau, ta có đẳng thức f (mn) = f (m)f (n). Những ví dụ đơn giản nhất về hàm nhân tính là f (n) = n và f (n) = 1. Ngoài các hàm nhân tính đơn giản kể trên thì ta phải kể đến các hàm nhân tính quan trọng khác như : Phi-hàm Euler, hàm sinh bởi các ước, hàm tổng
  • 10. Viết thuê đề tài giá rẻ trọn gói - KB Zalo/Tele : 0973.287.149 Luanvanmaster.com – Cần Kham Thảo - Kết bạn Zalo/Tele : 0973.287.149 các ước số, hàm số các ước số, hàm Mobius. Trong các hàm số học, hàm Euler mà ta định nghĩa sau đây có vai trò rất quan trọng.
  • 11. Viết thuê đề tài giá rẻ trọn gói - KB Zalo/Tele : 0973.287.149 Luanvanmaster.com – Cần Kham Thảo - Kết bạn Zalo/Tele : 0973.287.149 4 Định nghĩa 1.3 ([9]). Giả sử n là số nguyên dương. Phi-hàm Euler ϕ(n) là hàm số học có giá trị tại n bằng số các số không vượt quá n và nguyên tố cùng nhau với n. Ví dụ 1.1. Từ định nghĩa ta có ϕ(1) = 1, ϕ(2) = 1, ϕ(3) = 2, ϕ(4) = 2, ϕ(5) = 4, ϕ(6) = 2, ϕ(7) = 6, ϕ(8) = 4, ϕ(9) = 6, ϕ(10) = 4. Từ định nghĩa trên đây, ta có ngay hệ quả trực tiếp. Hệ quả 1.1 ([9]). Số p là nguyên tố khi và chỉ khi ϕ(p) = p − 1. Chứng minh. Nếu p là số nguyên tố thì với mọi số nguyên dương nhỏ hơn p đều nguyên tố cùng nhau với p. Do có p − 1 số nguyên dương như vậy nên ϕ(p) = p − 1. Ngược lại, nếu p là hợp số thì p có các ước d, 1 < d < p. Tất nhiên p và d không nguyên tố cùng nhau. Như vậy trong các số 1, 2, · · · , p − 1 phải có số không nguyên tố cùng nhau với p, nên ϕ(p) < p − 2. Điều này trái với giả thiết ϕ(p) = p − 1. Định nghĩa 1.4 ([2]). Một tập hợp A nào đó được gọi là một hệ thặng dư đầy đủ (mod n) chính là n số mà không có hai số nào đồng dư nhau theo (mod n). Ví dụ 1.2. Ta có A = {0, 1, 2, · · · , n − 1} là một hệ thặng dư đầy đủ theo (mod n). Hay đơn giản hơn tập B = {0, 1, 2, 3, 4, 5, 6, 7} và C = {2, 3, 4, 5, 6, −7, 8} là các hệ thặng dư đầy đủ theo (mod 8). Định nghĩa 1.5 ([2]). Một hệ thặng dư thu gọn môđulô n là tập hợp gồm ϕ(n) số nguyên sao cho mỗi phần tử của tập hợp đều nguyên tố cùng nhau với n và không có hai phần tử khác nhau nào đồng dư môđulô n. Ví dụ 1.3. Tập hợp {1, 3, 5, 7} và tập {3, −7, 5, 7} là các hệ thặng dư thu gọn (mod 8). Định lí 1.1 ([2]). Giả sử r1, r2, . . . , rϕ(n) là một hệ thặng dư thu gọn (mod n),
  • 12. Viết thuê đề tài giá rẻ trọn gói - KB Zalo/Tele : 0973.287.149 Luanvanmaster.com – Cần Kham Thảo - Kết bạn Zalo/Tele : 0973.287.149 a là số nguyên dương và (a, n) = 1. Khi đó tập ar1, ar2, · · · , arϕ(n) cũng là hệ thặng dư thu gọn (mod n).
  • 13. Viết thuê đề tài giá rẻ trọn gói - KB Zalo/Tele : 0973.287.149 Luanvanmaster.com – Cần Kham Thảo - Kết bạn Zalo/Tele : 0973.287.149 5 Chứng minh. Vì r1, r2, . . . , rϕ(n) là một hệ thặng dư thu gọn (mod n) nên tập 1 2 · · · ϕ(n) có ϕ(n) số. Ta chỉ cần chứng minh khi i = j thì ar 1 ∼ j ar, ar , , ar = ar . . (mod n). Thật vậy: Giả sử ari ≡ arj (mod n) suy ra a (ri − rj ) . n, vì (a, r) = 1 . . nên ri − rj . n suy ra ri ≡ rj (mod n), trái với r1, r2, . . . , rϕ(n) là hệ thặng dư thu gọn ( Vì hệ thặng dư thu gọn không có hai số nào đồng dư nhau). Ví dụ 1.4. Tập {1, 3, 5, 7 } là một hệ thặng dư thu gọn (mod 8). Do (3, 8) = 1 nên tập {3, 9, 15, 21} cũng là một hệ thặng dư thu gọn (mod 8) . Định lí 1.2 (Định lí Euler). (a, n) = 1. Khi đó aϕ(n) ≡ 1 Giả sử n là số nguyên dương và a là số nguyên với (mod n). Chứng minh. Xét hệ thặng dư thu gọn (mod n) r1, r2, · · · , rϕ(n) Suy ra ar1, ar2, · · · , arϕ(n) cũng là một hệ thặng dư thu gọn (mod n). Do đó ta có ar1.ar2 . . . arϕ(n) ≡ r1.r2 . . . rϕ(n)(mod n) ⇒ a ϕ(n) .r1.r2 . . . rϕ(n) ≡ r1.r2 . . . rϕ(n) (mod n) ϕ(n) . − 1 r1.r2 . ⇒ a . . . rϕ(n) . n ϕ(n) .. ⇒ a − 1 . n ⇒ a ϕ(n) ≡ 1(mod n). Chú ý 1.1. Ta có thể tìm nghịch đảo của (mod n) bằng cách sử dụng định lí Euler. Giả sử (a, m) = 1 khi đó a.aϕ(n)−1 = aϕ(n) ≡ 1(mod n) trong đó a ϕ(n)−1 là nghịch đảo của a(mod n). Ví dụ 1.5. 2 ϕ(9)−1 = 2 6−1 = 2 5 = 32 ≡ 5(mod 9) là ngịch đảo của 2(mod 9). Hệ quả 1.2 ([2]). Nếu (a, b) = 1 thì aϕ(b) + bϕ(a) ≡ 1(mod ab). Hệ quả 1.3 ([2]). Với (a, b) = 1 và n, v là hai số nguyên dương nào đó thì
  • 14. Viết thuê đề tài giá rẻ trọn gói - KB Zalo/Tele : 0973.287.149 Luanvanmaster.com – Cần Kham Thảo - Kết bạn Zalo/Tele : 0973.287.149 a nϕ(b) + b vϕ(a) ≡ 1 (modab) .
  • 15. Viết thuê đề tài giá rẻ trọn gói - KB Zalo/Tele : 0973.287.149 Luanvanmaster.com – Cần Kham Thảo - Kết bạn Zalo/Tele : 0973.287.149 6 Hệ quả 1.4 ([2]). Giả sử có k (k ≥ 2) số nguyên dương m1, m2, · · · , mk và chúng nguyên tố với nhau từng đôi một. Đặt M = m1m2 . . . mk = mi.ti với i = 1, 2, . . . , k ta có t n 1 + t n 2 + · · · + t n k ≡ (t1 + t2 + · · · + tk) n (mod M), với n nguyên dương. Định lí 1.3 ([2]). Phi-hàm Euler là hàm nhân tính. Chứng minh. Ta viết các số nguyên dương không vượt quá mn thành bảng sau: 1 m + 1 2m + 1 . . . (n − 1) m + 1 2 m + 2 2m + 2 . . . (n − 1) m + 2 3 m + 3 2m + 3 . . . (n − 1) m + 3 ... ... ...... . . . m 2m 3m . . . mn Bây giờ giả sử r là một số nguyên không vượt quá m. Giả sử (m, r) = d > 1. Khi đó, không có số nào trong dòng thứ r nguyên tố cùng nhau với mn, vì mỗi phần tử của dòng đó đều có dạng km + r, trong đó 1 ≤ k ≤ n − 1, d| (km + r), vì d|m, d|r. Vậy để các số trong bảng mà nguyên tố cùng nhau với mn, ta chỉ cần xét các dòng thứ r với (m, r) = 1. Ta xét một dòng như vậy, nó chứa các số r, m+r, . . . , (n − 1) m+r. Vì (m, r) = 1 nên mỗi số nguyên trong dòng đều nguyên tố cùng nhau với n. Như vậy n số nguyên trong dòng lập thành hệ thặng dư đầy đủ Môđulô n. Do đó có đúng ϕ(n) số trong hàng đó nguyên tố cùng nhau với n. Do các số đó cũng nguyên tố cùng nhau với m nên chúng nguyên tố cùng nhau với mn. Vì có ϕ(m) dòng, mỗi dòng chứa ϕ(n) số nguyên tố cùng nhau với mn nên ta suy ra ϕ(mn) = ϕ(m)ϕ(n). Định lí 1.4 ([2]). Giả sử n = p a 11 p a 22 . . . p a kk là phân tích n ra thừa số nguyên tố. Khi đó 1 1 1
  • 16. Viết thuê đề tài giá rẻ trọn gói - KB Zalo/Tele : 0973.287.149 Luanvanmaster.com – Cần Kham Thảo - Kết bạn Zalo/Tele : 0973.287.149 ϕ(n) = n 1 − 1 − ... 1− . p1 p2 pk
  • 17. Viết thuê đề tài giá rẻ trọn gói - KB Zalo/Tele : 0973.287.149 Luanvanmaster.com – Cần Kham Thảo - Kết bạn Zalo/Tele : 0973.287.149 7 Chứng minh. Vì ϕ là hàm có tính chất nhân nên nếu n có phân tích như trên thì ϕ (n) = ϕ (p a 11 ) ϕ (p a 22 ) . . . ϕ (p a kk ) . Mặt khác ϕ pj aj = pj aj − pj aj −1 = pj aj 1 1 − , j = 1, 2 . . . , k pj Vậy 1 1 . . . pk ak 1 ϕ (n) = p1 a1 1 − p2 a2 1− 1 − p1 p2 pk = p1 a1 p2 a2 . . . pk ak 1 1 1 1 − 1 − ... 1− p 1 p 2 p k 1 1 1 = n 1 − 1 − ... 1− . p1 p2 pk Định lí 1.5 ([2]). Giả sử n là số nguyên dương. Khi đó ϕ(d) = n, d|n trong đó tổng được lấy theo mọi ước của n. Chứng minh. Tổng trên đây được lấy theo các ước của n. Ta phân chia tập hợp các số tự nhiên từ 1 đến n thành các lớp sau đây. Lớp Cd gồm các số nguyên m, 1 ≤ m ≤ n, mà (m, n) = d. Như vậy m thuộc lớp Cd nếu và chỉ nếu d là ước chung của m, n và (m/d, n/d) = 1. Như vậy, số phần tử của lớp Cd là số nguyên dương không vượt quá n/d và nguyên tố cùng nhau với n/d; tức là Cd gồm ϕ(n/d) phần tử. Vì mỗi số nguyên m từ 1 đến n thuộc một và chỉ một lớp Cd nào đó (d = (m, n)) nên n bằng tổng của số các thành phần trong các lớp Cd, d là ước số của n. Ta có n = ϕ n . d|n d Bên cạnh Phi-hàm Euler thì ta cần nghiên cứu đến lớp các hàm sinh bởi các ước, trước tiên ta tìm hiểu về hàm tổng các ước. Định nghĩa 1.6 ([2]). Hàm tổng các ước dương của số tự nhiên n được kí hiệu là σ(n). Ví dụ 1.6. σ(12) = 1 + 2 + 3 + 4 + 6 + 12 = 28.
  • 18. Viết thuê đề tài giá rẻ trọn gói - KB Zalo/Tele : 0973.287.149 Luanvanmaster.com – Cần Kham Thảo - Kết bạn Zalo/Tele : 0973.287.149 Chú ý 1.2. Ta có thể biểu diễn σ(n) dưới dạng σ(n) = d. d|n
  • 19. Viết thuê đề tài giá rẻ trọn gói - KB Zalo/Tele : 0973.287.149 Luanvanmaster.com – Cần Kham Thảo - Kết bạn Zalo/Tele : 0973.287.149 8 Bổ đề 1.1 ([2]). Giả sử m, n là các số nguyên dương nguyên tố cùng nhau. Khi đó nếu d là ước chung của mn thì tồn tại cặp duy nhất các ước dương d1 của m và d2 của n sao cho d = d1d2. Ngược lại, nếu d1, d2 là ước dương tương ướng của m và n thì d = d1d2 là ước dương của mn. Chứng minh. Giả sử m, n có phân tích ra thừa số nguyên tố như sau m = p m 1 1 p m 2 2 . . . p m s s , n = q1 n1 q2 n2 . . . qt nt . Vì (m, n) = 1 nên tập hợp các số nguyên tố p1, p2, . . . , ps và tập hợp các số nguyên tố q1, q2, . . . , qt không có phần tử chung. Do đó phân tích ra thừa số của mn có dạng mn = pm 11 pm 22 . . . pm ss .q1 n 1 q2 n 2 . . . qt n t . Như vậy, nếu d là một ước chung của mn thì d = pe 1 1 pe 2 2 . . . pe ss .q1 f1 q2 f2 . . . qt ft , trong đó 0 ≤ ei ≤ mi (i = 1, 2, . . . , s) ; 0 ≤ fi ≤ ni (i = 1, 2, . . . , t) . Đặt d1 = pe 1 1 pe 2 2 . . . pe ss , d2 = q1 f1 q2 f2 . . . qt ft . Rõ ràng d = d1d2 và (d1, d2) = 1. Ngược lại, giả sử d1 và d2 là các ước dương tương ứng của m và n khi đó d1 = p e 11 pe 22 . . . pe ss trong đó 0 ≤ ei ≤ mi (i = 1, 2, . . . , s) và d2 = q1 f1 q2 f2 . . . qt ft trong đó 0 ≤ fi ≤ ni (i = 1, 2, . . . , t) . Số nguyên d = d1d2 = pe 1 1 pe 2 2 . . . pe ss .q1 f1 q2 f2 . . . qt ft là ước của mn = pm 11 pm 22 . . . pm ss .q1 n 1 q2 n 2 . . . qt n t . Vì lũy thừa của mỗi số nguyên tố xuất hiện trong phân tích ra thừa số nguyên tố của d bé hơn hoặc bằng lũy thừa của số nguyên tố đó trong phân tích của mn. Bổ đề 1.2 ([2]). Giả sử p nguyên tố a nguyên dương. Khi đó: σ(pa) = 1 + p + p 2 + · · · + pa = pa − 1 p − 1 τ (p a ) = a + 1 Chứng minh. Các ước của pa là 1, p, p2 , . . . , pa . Do đó pa có đúng a + 1 ước dương. Suy ra τ (p a ) = a + 1 và σ(p a ) = 1 + p + p 2 + · · · + p a = pa − 1 . p − 1
  • 20. Viết thuê đề tài giá rẻ trọn gói - KB Zalo/Tele : 0973.287.149 Luanvanmaster.com – Cần Kham Thảo - Kết bạn Zalo/Tele : 0973.287.149 Định lí 1.6 ([9]). Giả sử f là hàm có tính chất nhân. Khi đó hàm F (n) = f (d) d|n cũng có tính chất nhân.
  • 21. Viết thuê đề tài giá rẻ trọn gói - KB Zalo/Tele : 0973.287.149 Luanvanmaster.com – Cần Kham Thảo - Kết bạn Zalo/Tele : 0973.287.149 9 Chứng minh. Ta sẽ chỉ ra rằng nếu m, n là các số nguyên dương nguyên tố cùng nhau thì F (mn) = F (m).F (n). Giả sử (m, n) = 1, ta có F (mn) = f (d). Vì d|mn (m, n) = 1 nên theo bổ đề 1.1, mỗi ước của mn có thể viết duy nhất dưới dạng tích các ước d1 của m và d2 của n và d1, d2 nguyên tố cùng nhau, đồng thời mỗi cặp ước số d1 của m và d2 của n tương ứng với ước d1.d2 của mn. Do đó ta có thể viết F (mn) = f (d1) f (d2) = f (d1) . f (d2) = F (m) F (n) . d1 d2 |m |n d1 |m d2|n Từ định lí 1.6 và bổ đề 1.2 ta suy ra trực tiếp tính chất sau đây. Tính chất 1.1. σ(n) là hàm nhân tính. Tính chất 1.2. Nếu p nguyên tố thì σ(p) = p + 1. Tính chất 1.3. Giả sử n là số nguyên dương và có khai triển chính tắc n = pα1 pα2 . . . p α k thì σ(n) = p1 α 1 +1 − 1 .p2 α 2 +1 − 1 . . .pk α k+1 − 1 1 2 k p1 − 1p2 − 1 pk − 1 Chứng minh. Do hàm σ là hàm nhân tính nên ta có σ(n) = σ(p α1 )σ(p α2 ) . . . σ(pαs ) 1 2 s α1 α2 αk ) = (1 + p1 + · · · + p1 ) (1 + p2 + · · · + p2 ) . . . (1 + pk + · · · + pk = p1 α 1 +1 − 1 .p2 α 2 +1 − 1 . . .pk α k+1 − 1 p1 − 1 pk − 1 p2 − 1 Trong lớp hàm sinh bởi các ước thì ta cũng phải kể đến hàm số các ước được định nghĩa như sau. Định nghĩa 1.7 ([2]). Số các ước dương của số tự nhiên n được kí hiệu là τ (n). Ví dụ 1.7. τ (1) = 1, τ (2) = 2, τ (12) = 6. Định lí 1.7 ([9]). Hàm τ (n) là hàm nhân tính.
  • 22. Viết thuê đề tài giá rẻ trọn gói - KB Zalo/Tele : 0973.287.149 Luanvanmaster.com – Cần Kham Thảo - Kết bạn Zalo/Tele : 0973.287.149 Chứng minh. Xét hàm f (n) ≡ 1 với mọi số tự nhiên n. Khi đó f (n) là hàm nhân tính. Gọi d1, d2, . . . , dk là tất cả các ước dương của n (kể cả 1 và n). Khi đó τ (n) = k = f (d1) + f (d2) + · · · + f (dk) . Vậy τ (n) là hàm nhân tính.
  • 23. Viết thuê đề tài giá rẻ trọn gói - KB Zalo/Tele : 0973.287.149 Luanvanmaster.com – Cần Kham Thảo - Kết bạn Zalo/Tele : 0973.287.149 10 Định lí 1.8 ([9]). Nếu p là nguyên tố thì τ (p) = 2. Chứng minh.Vì p nguyên tố nên p chỉ có hai ước dương là 1 và p. Do đó τ(p) = 2. Định lí 1.9 ([9]). Nếu số nguyên dương n có khai triển ra thừa số nguyên tố dạng n = p α 1 1 p α 2 2 . . . p α kk thì τ (n) = (α1 + 1) (α2 + 1) . . . (αk + 1) . Chứng minh. Vì τ (n) là hàm nhân tính nên ta có τ (n) = τ (p α 1 1 ) τ (p α 2 2 ) . . . τ (p α kk ) = (α1 + 1) (α2 + 1) . . . (αk + 1) , bởi vì p α ii có αi + 1 ước dương với i = 1, 2, . . . , k. Ngoài các hàm nhân tính kể trên thì ta cũng cần xét đến hàm nhân tính sau Định nghĩa 1.8 (Hàm Mobius). Hàm Mobius là hàm số học xác định như sau: 1 nếu n = 1; | µ(n) = 0 nếu p 2 n đối với số nguyên tố p > 1; 2 n = p1p2 . . . pk ở đó , . . . , pk là các số nguyên tố phân biệt. (−1)knếu p 1 Ví dụ 1.8. µ(2) = −1, µ(6) = 1, µ(12) = µ(2 .3) = 0. Định lí 1.10 ([9]). Hàm Mobius là hàm nhân tính. Chứng minh. Giả sử m, n là những số nguyên dương sao cho gcl (m, n) = 1. Nếu p 2 |m với moi p > 1 thì p 2 |mn và µ (m) = µ (mn) = 0 và khẳng định trên là đúng. Bây giờ xét m = p1.p2 . . . pk và n = q1.q2 . . . qh với p1, p2, . . . , pk và q1, q2, . . . , qh là các số nguyên tố phân biệt. Khi đó µ (m) = (−1) k , µ (n) = (−1) h và mn = p1.p2 . . . pk.q1.q2 . . . qh. Suy ra µ (mn) = (−1)k+h = (−1)k (−1)h = µ (m) .µ (n) . Định lí 1.11 ([9]). Cho f là một hàm số học và F (n) = f (d). Khi đó d|n f (n) = µ (d) F n .
  • 24. Viết thuê đề tài giá rẻ trọn gói - KB Zalo/Tele : 0973.287.149 Luanvanmaster.com – Cần Kham Thảo - Kết bạn Zalo/Tele : 0973.287.149 d d|n
  • 25. Viết thuê đề tài giá rẻ trọn gói - KB Zalo/Tele : 0973.287.149 Luanvanmaster.com – Cần Kham Thảo - Kết bạn Zalo/Tele : 0973.287.149 11 Chứng minh. Ta có n c| c| n µ(d) f (c) d|n µ (d) F d = d|n µ (d) n f (c) = d|n d d d| = c|n n µ (d) f (c) = c d| c|n f (c) n µ (d) = f (n) . c Từ n > 1 ta có n µ(d) = 0. Thực tế các tập (d, c) |d|n, c| n và (d, c) |c|n, d| n c d d d| c là bằng nhau. 1.1.2 Hàm nhân tính mạnh Định nghĩa 1.9 ([9]). Một hàm số học f được gọi là nhân tính mạnh nếu với mọi m, n (không nhất thiết nguyên tố cùng nhau), ta có đẳng thức f (mn) = f (m)f (n). Những ví dụ đơn giản nhất về hàm nhân tính mạnh là f (n) = n và f (n) = 1. 1.2 Hàm số xác định trên tập các số nguyên 1.2.1 Hàm cộng tính trên tập các số nguyên Định nghĩa 1.10 ([9]). Một hàm số f (x) xác định trên tập số nguyên được gọi là hàm cộng tính nếu với mọi m, n ta có đẳng thức f (m + n) = f (m) + f (n). 1.2.2 Hàm nhân tính trên tập các số nguyên Định nghĩa 1.11 ([9]). Một hàm số f (x) xác định trên tập số nguyên được gọi là hàm nhân tính nếu với mọi m, n ta có đẳng thức
  • 26. Viết thuê đề tài giá rẻ trọn gói - KB Zalo/Tele : 0973.287.149 Luanvanmaster.com – Cần Kham Thảo - Kết bạn Zalo/Tele : 0973.287.149 f (mn) = f (m)f (n).
  • 27. Viết thuê đề tài giá rẻ trọn gói - KB Zalo/Tele : 0973.287.149 Luanvanmaster.com – Cần Kham Thảo - Kết bạn Zalo/Tele : 0973.287.149 12 1.2.3 Lớp hàm tuần hoàn, phản tuần hoàn cộng tính, nhân tính Định nghĩa 1.12 ([5]). Cho hàm số f (x) xác định trên tập số nguyên. Hàm f (x) được gọi là hàm tuần hoàn trên Z nếu tồn tại số nguyên dương a sao cho ∀x ∈ Z ta đều có x ± a ∈ Z f (x + a) = f (x) , ∀x ∈ Z a được gọi là chu kỳ của hàm tuần hoàn f (x). Chu kỳ nhỏ nhất (nếu có) trong các chu kỳ của f (x) được gọi là chu kỳ cơ sở của hàm tuần hoàn f (x). Ví dụ 1.9. Cho cặp hàm f (x), g(x) xác định trên tập số nguyên, tuần hoàn trên Z và có chu kỳ lần lượt là a và b (a, b ∈ Z+ ) với a ∈ Q+ . Chứng minh rằng b Lời giải. Theo giả thiết tồn tại m, n ∈ Z + , (m, n) = 1 sao cho a b = m n. Đặt T = na = mb. Khi đó T F (x + T ) = f (x + na) + g(x + mb) = f (x) + g(x) = F (x) G(x + T ) = f (x + na)g(x + mb) = f (x)g(x) = G(x) Hơn nữa, dễ thấy ∀x ∈ Z thì x ± T ∈ Z. Vậy F (x) và G(x) là những hàm tuần hoàn trên Z. Ví dụ 1.10. Xét hàm số f (x) = sin 2 3 π x xác định trên Z. Chứng minh rằng f (x) là hàm tuần hoàn với chu kỳ T = 3. Lời giải. Ta có ∀x ∈ Z thì x ± 3 ∈ Z và 2π 2π f (x + 3) = sin 3 (x + 3) = sin 3 x + 2π = f (x). Định nghĩa 1.13 ([5]). Cho hàm số f (x) xác định trên tập số nguyên. Hàm f (x) được gọi là hàm phản tuần hoàn trên Z nếu tồn tại số nguyên dương a sao cho
  • 28. Viết thuê đề tài giá rẻ trọn gói - KB Zalo/Tele : 0973.287.149 Luanvanmaster.com – Cần Kham Thảo - Kết bạn Zalo/Tele : 0973.287.149 ∀x ∈ Z ta đều có x ± a ∈ Z f (x + a) = −f (x) , ∀x ∈ Z
  • 29. Viết thuê đề tài giá rẻ trọn gói - KB Zalo/Tele : 0973.287.149 Luanvanmaster.com – Cần Kham Thảo - Kết bạn Zalo/Tele : 0973.287.149 13 a được gọi là chu kỳ của hàm phản tuần hoàn f (x). Chu kỳ nhỏ nhất (nếu có) trong các chu kỳ của f (x) được gọi là chu kỳ cơ sở của hàm phản tuần hoàn f (x). Ví dụ 1.11. Chứng minh rằng mọi hàm phản tuần hoàn xác định trên tập số nguyên cũng là hàm tuần hoàn xác định trên tập số nguyên. Lời giải. Theo giả thiết tồn tại số nguyên dương b sao cho ∀x ∈ Z thì x ± b ∈ Z và f (x + b) = −f (x), ∀x ∈ Z. Suy ra ∀x ∈ Z thì x ± 2b ∈ Z và f (x + 2b) = f (x + b + b) = −f (x + b) = f (x), ∀x ∈ Z. Vậy f (x) là hàm tuần hoàn với chu kỳ 2b trên Z. Ví dụ 1.12. Xét hàm số f (x) = sin π 3 x xác định trên Z. Chứng minh rằng f (x) là hàm phản tuần hoàn với chu kỳ T = 3. Lời giải. Ta có ∀x ∈ Z thì x ± 3 ∈ Z và π π f (x + 3) = sin 3 (x + 3) = sin 3 x + π = −f (x). Định nghĩa 1.14 ([5]). Hàm f (x) xác định trên tập số nguyên được gọi là tuần hoàn nhân tính chu kỳ nguyên dương a (a ∈/ {−1, 0, 1}) trên Z nếu ∀x ∈ Z ⇒ a ±1 x ∈ Z f (ax) = f (x) , ∀x ∈ Z Ví dụ 1.13. Xét hàm số f (x) = sin (2πlog2x) xác định trên Z + . Khi đó f (x) là hàm tuần hoàn nhân tính chu kỳ 2. Thật vậy, ta có ∀x ∈ Z + thì 2 ±1 x ∈ Z + và f (2x) = sin (2πlog2(2x))
  • 30. Viết thuê đề tài giá rẻ trọn gói - KB Zalo/Tele : 0973.287.149 Luanvanmaster.com – Cần Kham Thảo - Kết bạn Zalo/Tele : 0973.287.149 = sin (2π (1 + log2x)) = sin (2πlog2x) = f (x), ∀x ∈ Z+ .
  • 31. Viết thuê đề tài giá rẻ trọn gói - KB Zalo/Tele : 0973.287.149 Luanvanmaster.com – Cần Kham Thảo - Kết bạn Zalo/Tele : 0973.287.149 14 Định nghĩa 1.15 ([5]). Hàm f (x) xác định trên tập số nguyên được gọi là hàm phản tuần hoàn nhân tính chu kỳ dương a (a ∈/ {−1, 0, 1}) trên Z nếu ∀x ∈ Z ⇒ a ±1 x ∈ Z f (ax) = −f (x) , ∀x ∈ Z Ví dụ 1.14. Xét hàm số f (x) = sin (πlog2x) xác định trên Z + . Khi đó f (x) là hàm phản tuần hoàn nhân tính chu kỳ 2. Thật vậy, ta có ∀x ∈ Z + thì 2 ±1 x ∈ Z + và f (2x) = sin (πlog2(2x)) = sin (π (1 + log2x)) = − sin (πlog2x) = −f (x), ∀x ∈ Z+ . 1.3 Một số bài tập áp dụng Bài toán 1.1. Tìm tất cả các số tự nhiên n có tính chất n chia hết cho ϕ(n). Lời giải. Ta xét các trường hợp sau. - Nếu n = 1 thì ϕ(n)|n. - Nếu n > 1, giả sử n có phân tích thành thừa số nguyên tố là n = p k 11 p k 22 . . . p k ii . Khi đó, ta có 1 1 1 ϕ(n) = n 1 − 1 − ... 1− . p1 p2 pi Từ điều kiện n = xϕ(n) ta suy ra p1p2 . . . pi = x (p1 − 1) (p2 − 1) . . . (pi − 1) . Điều này có nghĩa là phải có pj nào đó bằng 2 (vì nếu ngược lại thì sẽ dẫn đến vô lí vì vế trái là số lẻ, trong khi vế phải là số chẵn). Do p2, . . . , pi khác 2 nên từ trên suy ra rằng n có nhiều nhất một ước nguyên tố lẻ, chẳng hạn p2. Đặt p2 = 2y + 1. Ta có 2p2 = x(2y). Do p2 là nguyên tố nên x = p2, y = 1. Như vậy p2 = 3 và n có dạng n = 2 k 2 m với k ≥ 1 và m ≥ 0.
  • 32. Viết thuê đề tài giá rẻ trọn gói - KB Zalo/Tele : 0973.287.149 Luanvanmaster.com – Cần Kham Thảo - Kết bạn Zalo/Tele : 0973.287.149 Thử lại thấy n = 2 k 2 m với k ≥ 1, m ≥ 0 thỏa mãn.
  • 33. Viết thuê đề tài giá rẻ trọn gói - KB Zalo/Tele : 0973.287.149 Luanvanmaster.com – Cần Kham Thảo - Kết bạn Zalo/Tele : 0973.287.149 15 Bài toán 1.2. Chứng minh rằng ϕ(n) là chẵn nếu n = 1 hoặc n = 2 và lẻ nếu với mọi n ≥ 3. Lời giải. Ta xét các trường hợp sau. - Với n = 1 hoặc n = 2 ta có ϕ(1) = ϕ(2) = 1 lẻ. - Với n ≥ 3 và n = p α 11 p α 22 . . . p α rr . ta có ϕ(n) = p α 1 1−1 p α 2 2−1 . . . p α rr −1 (p1 − 1) (p2 − 1) . . . (pr − 1) . Vì ít nhất một trong các số hạng p1, p2, . . . , pr, p1 − 1, p2 − 1, . . . , pr − 1 là chẵn nên ϕ(n) là chẵn. Bài toán 1.3 (Canada 1999). Tìm tất cả các số nguyên dương n sao cho n = τ 2 (n). Lời giải. Giả sử n là số nguyên dương thỏa mãn yêu cầu bài toán. k Do n = τ 2 (n) là một số chính phương nên ta có thể đặt n = i=1 τ (n) = k (2αi + 1) i i=1 k 2α +1 i=1 pi αi = 1 Áp dụng bất đẳng thức Bernoulli ta có p α i i ≥ (pi − 1) αi + 1 > 2αi + 1. Với mọi ước số nguyên tố pi ≥ 5 của n. Mặt khác, ta có 3 α ≥ 2α + 1 và dấu bằng xảy ra khi và chỉ khi α ∈ {0, 1}. Suy ra n = 1, n = 9 thỏa mãn yêu cầu bài toán. Bài toán 1.4. Tìm các số nguyên dương n sao cho σ(n) = 12. Lời giải. Để giải quyết tốt bài toán này ta cần chọn lựa các số nguyên tố và số mũ thích hợp sao cho: (1 + p1 + · · · + p α 11 ) (1 + p2 + · · · + p α 22 ) . . . (1 + pk + · · · + p α kk ) = σ(n). Liệt kê các số có dạng 1 + pk + p 2 k + · · · + p α kk từ nhỏ đến lớn p2 i α i . Khi đó
  • 34. Viết thuê đề tài giá rẻ trọn gói - KB Zalo/Tele : 0973.287.149 Luanvanmaster.com – Cần Kham Thảo - Kết bạn Zalo/Tele : 0973.287.149 1+2;1+3;1+5;1+2+2 2 ;1+7;1+3+3 2 ;1+2+2 2 +2 3 ,...
  • 35. Viết thuê đề tài giá rẻ trọn gói - KB Zalo/Tele : 0973.287.149 Luanvanmaster.com – Cần Kham Thảo - Kết bạn Zalo/Tele : 0973.287.149 16 Với σ(n) = 12 ta thấy chỉ có thể viết thành 12 = (1 + 2)(1 + 3) và số cần tìm là n = 6. Bài toán 1.5. Với giá trị nào của n thì σ(n) là số lẻ Lời giải. Cần chọn các ước số nguyên tố và số mũ thích hợp để σ(n) là số lẻ. Ta có σ(n) = (1 + p1 + · · · + pα 1 1 ) (1 + p2 + · · · + pα 2 2 ) . . . (1 + pk + · · · + pα kk ) . Khi đó nếu có ước nguyên tố nào đó là 2 thì vẫn thỏa mãn; nếu n có ước nguyên tố lẻ thì trong tổng trên chúng phải xuất hiện chẵn lần. Do đó số cần tìm có dạng n = 2 k m 2 với m là số chính phương lẻ. Bài toán 1.6. Chứng minh rằng điều kiện cần và đủ để n là một hợp số là: σ(n) + 1 4 > √ n + 1 2. Lời giải. Trước hết, ta có σ(n) + 1 4 > √ n + 1 2 ⇔σ(n) + 1 4 > n + √ n + 1 4 ⇔σ(n) > n + √ n. Phần thuận Giả sử n là hợp số. Khi đó ngoài các ước 1 và n thì n còn ít nhất một ước là d với 1 < d < n. Lúc này n d cũng là một ước của n ( dễ thấy 1 < n d < n). Xét hai trường hợp sau: - Nếu n d = d thì sẽ có ít nhất 4 ước là 1, n, d, n d. Khi đó σ(n) ≥ 1 + n + d + n ≥ 1 + n + 2 n √ √ d. = 1 + n + 2 n > n + n. d d - Nếu n d = d tức d = √ n thì lúc này n có ít nhất 3 ước là 1, √ n, n. Vì thế σ(n) ≥ 1 + n + √ n > n + √ n. Như vậy khi n là hợp số thì σ(n) > n + √ n. Phần đảo Giả sử σ(n) > n + √ n. Vì σ(1) = 1 < 1 + √ 1 vô lý, cho nên n = 1. Hơn nữa n
  • 36. Viết thuê đề tài giá rẻ trọn gói - KB Zalo/Tele : 0973.287.149 Luanvanmaster.com – Cần Kham Thảo - Kết bạn Zalo/Tele : 0973.287.149 không thể là số nguyên tố. Thật vậy, nếu p là số nguyên tố thì σ(p) = p + 1 < p + √ p ( chú ý p ≥ 2 ), lại là điều vô lý.
  • 37. Viết thuê đề tài giá rẻ trọn gói - KB Zalo/Tele : 0973.287.149 Luanvanmaster.com – Cần Kham Thảo - Kết bạn Zalo/Tele : 0973.287.149 17 Tóm lại, nếu n thỏa mãn bất đẳng thức σ(n) > n + √ n. thì n phải khác 1 và không phải số nguyên tố. Điều này đồng nghĩa với n phải là hợp số. Vậy ta hoàn tất chứng minh. Bài toán 1.7. Ký hiệu d(n) là trung bình cộng của tất cả các ước số của n ( kể cả 1 và n). Chứng minh rằng với mọi n ta có bất đẳng thức √ n + 1 n ≤ d(n) ≤ . 2 Lời giải. Giả sử 1 = d1 < d2 < · · · < dk = n là tất cả các ước của n. Như vậy 1 ≤ di ≤ n với i = 1, 2, . . . , n. Theo định nghĩa ta có d(n) = d1 + d2 + · · · + dk . n n n k Để ý rằng > > ··· > > n và chúng cũng là tất cả các ước của n. Do d1 d2 dk−1 dk đó n n n n d1 = , d2 = , . . . , dk−1 = , dk = . dk dk−1 d2 d1 Vì thế ta có d1dk = d2dk−1 = · · · = d1dk−i+1 = n với i = 1, 2, . . . , k − 1. (1.1) Ta có 2(d1 + d2 + · · · + dk) d1 + d2 +d2 + dk−1 +... dk−1 + d2 +dk + d1 d(n) = = 2 2 2 2 . k 2k Áp dụng bất đẳng thức AM - GM ta có: √ + √ +···+ √ + √ . d(n) d1dk d2dk−1 dk−1d2 dkd1 (1.2) ≥ k √ Từ (1.1) và (1.2) ta suy ra d(n) ≥ k n = √ n. Chứng minh xong bất đẳng thức thứ k nhất. Ta sẽ chứng minh bất đẳng thức còn lại. di ≥ 1 với mọi i = 1, 2, . . . , k; Nên
  • 38. Viết thuê đề tài giá rẻ trọn gói - KB Zalo/Tele : 0973.287.149 Luanvanmaster.com – Cần Kham Thảo - Kết bạn Zalo/Tele : 0973.287.149 0 ≤ (di − 1) (dk−i+1 − 1) = didk−i+1 + 1 − di − dk−i+1. (1.3)
  • 39. Viết thuê đề tài giá rẻ trọn gói - KB Zalo/Tele : 0973.287.149 Luanvanmaster.com – Cần Kham Thảo - Kết bạn Zalo/Tele : 0973.287.149 18 Từ (1.1) và (1.3) suy ra n + 1 − (di + dk−i+1) ≥ 0, với mọi i = 1, 2, . . . , k (1.4) Cộng vế theo vế k bất đẳng thức dạng (1.4) ta được: k 1 k n + 1 2 i=1 d i ≤ k (n + 1) ⇔ i=1 d i ≤ k 2 Chứng minh xong bất đẳng thức còn lại. Vậy ta hoàn tất việc chứng minh bất đẳng thức kép ở trên. Bài toán 1.8 (Korean MO 1998). Cho n là một số nguyên dương, ký hiệu ψ(n) là số các thừa số nguyên tố của n. Chứng minh rằng nếu ϕ(n) chia hết cho n − 1 và ψ(n) ≤ 3 thì n là số nguyên tố. Lời giải. Để ý rằng với số nguyên tố p, nếu p2 |n thì p|ϕ(n) nhưng p không là ước của n − 1, điều này mâu thuẫn. Vì vậy chúng ta chỉ cần chứng tỏ rằng n = pq, n = pqr với các số nguyên tố p < q < r. Trước hết, giả sử n = pq, vì vậy (p − 1)(q − 1)|pq − 1 Chú ý rằng q ≥ 3 suy ra vế trái là số chẵn, cho nên vế phải cũng vậy và p, q là các số lẻ. Xét trường hợp p = 3, q = 5. Khi đó pq − 1 < 2. (1.5) (p − 1)(q − 1) Vế trái của (1.5) là một hàm tăng với từng biến và luôn lơn hơn 1, vì vậy nó không thể là số nguyên. Đây là điều mâu thuẫn. Bây giờ, xét n = pqr. Như trên p, q, r là lẻ ; nếu p = 3, q = 5 và r = 11 thì pqr − 1 < 2. (1.6) (p − 1)(q − 1)(r − 1)
  • 40. Viết thuê đề tài giá rẻ trọn gói - KB Zalo/Tele : 0973.287.149 Luanvanmaster.com – Cần Kham Thảo - Kết bạn Zalo/Tele : 0973.287.149 Vế trái của (1.6) là một hàm tăng với từng biến và luôn lơn hơn 1, vì vậy nó không thể là số nguyên. Đây là điều mâu thuẫn.
  • 41. Viết thuê đề tài giá rẻ trọn gói - KB Zalo/Tele : 0973.287.149 Luanvanmaster.com – Cần Kham Thảo - Kết bạn Zalo/Tele : 0973.287.149 19 Điều này giúp loại bỏ các trường hợp ngoại trừ p = 3, q = 5 và r = 7 ta có pqr − 1 < 3. (1.7) (p − 1)(q − 1)(r − 1) Vì vế trái của (1.7) chỉ nhận giá trị nguyên nên pqr − 1 = 2. (p − 1)(q − 1)(r − 1) Để ý rằng 15r − 1 = 2 cho ta r = 15 không phải là số nguyên tố và ta loại bỏ 8(r − 1) tất cả các trường hợp.
  • 42. Viết thuê đề tài giá rẻ trọn gói - KB Zalo/Tele : 0973.287.149 Luanvanmaster.com – Cần Kham Thảo - Kết bạn Zalo/Tele : 0973.287.149 20 Chương 2 Các phương trình hàm số học Trong chương này sẽ trình bày về hàm chuyển đổi các phép tính số học, các dạng toán xác định dãy số liên quan và các bài tập áp dụng. 2.1 Hàm chuyển đổi các phép tính số học Trong mục này, ta giải các bài toán xác định các hàm chuyển đổi các phép tính số học đơn giản của đối số sang phép tính đối với các giá trị hàm tương ứng. Ta sẽ giải quyết các bài toán trên lớp hàm xác định trên tập số nguyên dương. 2.1.1 Hàm chuyển đổi phép cộng thành phép cộng Bài toán 2.1. Xác định tất cả các hàm số f : N∗ → R thỏa mãn f (x + y) = f (x) + f (y), ∀x, y ∈ N ∗ . (2.1) Lời giải. Từ (2.1) suy ra với x = y thì f (2x) = 2f (x), ∀x ∈ N ∗ Giả sử với k nguyên dương, f (kx) = kf (x), ∀x ∈ N ∗ . Khi đó f ((k + 1) x) = f (kx) + f (x) = kf (x) + f (x) = (k + 1) f (x) , ∀x ∈ N ∗ , ∀k ∈ N ∗ . Từ đó, theo nguyên lí quy nạp, ta có
  • 43. Viết thuê đề tài giá rẻ trọn gói - KB Zalo/Tele : 0973.287.149 Luanvanmaster.com – Cần Kham Thảo - Kết bạn Zalo/Tele : 0973.287.149 f (nx) = nf (x), ∀x ∈ N ∗ , ∀n ∈ N ∗
  • 44. Viết thuê đề tài giá rẻ trọn gói - KB Zalo/Tele : 0973.287.149 Luanvanmaster.com – Cần Kham Thảo - Kết bạn Zalo/Tele : 0973.287.149 21 Tóm lại ta có f (nx) = nf (x), ∀x ∈ N ∗ ∀n ∈ N ∗ . Trong quan hệ trên cho x = 1 ta được f (n) = cn, ∀n ∈ N∗ (c = f (1) là hằng số). Thử lại ta thấy hàm này thỏa mản bài ra. Nhận xét. - Kết quả của bài toán vẫn đúng khi ta mở rộng tập nguồn và tập đích lên tập Z. - Kết quả của bài toán rất quan trọng, được dùng để giải nhiều phương trình hàm trên Z. 2.1.2 Hàm chuyển đổi phép cộng thành phép nhân Bài toán 2.2. Xác định tất cả các hàm số f : N ∗ → R thỏa mãn f (x + y) = f (x)f (y), ∀x, y ∈ N∗ . (2.2) Lời giải. Với x chẵn ta thu được f (x) = f (2m) = [f (m)] 2 ≥ 0, ∀m ∈ N ∗ . Xét các trường hợp: - Nếu f (1) = 0, thì f (n + 1) = f (1)f (n) = 0 nên f (x) ≡ 0 và hàm này thỏa mãn điều kiện (2.2) nên là một nghiệm của bài toán. - Xét trường hợp f (1) = a > 0. Khi đó f (2) = [f (1)2 ] = a2 > 0 và f (3) = f (1)f (2 + 1) = a 3 > 0. Bằng phương pháp quy nạp, ta chứng minh được f (n) = a n với mọi n ∈ N ∗ . - Xét trường hợp f (1) = b < 0. Khi đó f (2) = [f (1) 2 ] = b 2 > 0 và f (3) = f (1)f (2 + 1) = b 3 < 0. Bằng phương pháp quy nạp, ta chứng minh được f (n) = b n với mọi n ∈ N ∗ . Kết luận.
  • 45. Viết thuê đề tài giá rẻ trọn gói - KB Zalo/Tele : 0973.287.149 Luanvanmaster.com – Cần Kham Thảo - Kết bạn Zalo/Tele : 0973.287.149 Vậy f (x) ≡ 0 và f (x) = a x , với a = 0.
  • 46. Viết thuê đề tài giá rẻ trọn gói - KB Zalo/Tele : 0973.287.149 Luanvanmaster.com – Cần Kham Thảo - Kết bạn Zalo/Tele : 0973.287.149 22 2.1.3 Hàm chuyển đổi phép nhân thành phép cộng Bài toán 2.3. Xác định tất cả các hàm số f : N ∗ → R và thỏa mãn f (xy) = f (x) + f (y), ∀x, y ∈ N ∗ . (2.3) Lời giải. Từ (2.3), ta thu được ef (xy) = ef (x)ef (y), ∀ x, y ∈ N∗ . (2.4) Đăt ef (t) = g(t), thì g(x) > 0, ∀t ∈ N∗ và (2.4) có dạng g(xy) = g(x)g(y), ∀x, y ∈ N ∗ . Vậy g(t) là hàm dương nhân tính mạnh trên N ∗ . Suy ra f (x) = ln g(t) trong đó g(t) là hàm dương nhân tính mạnh tùy ý trên N∗ . 2.2 Các dạng toán xác định dãy số liên quan Bài toán phương trình hàm trên N trên một phương diện nào đó, có thể coi là bài toán dãy số. Vì vậy, dưới đây chúng ta sẽ xét một số bài toán xác định dãy số liên quan. Bài toán 2.4 (IMO 1983). Chứng minh hoặc phủ định mệnh đề sau: Từ tập hợp 10 5 số nguyên dương đầu tiên luôn có thể chọn ra một tập con gồm 1983 số sao cho không có ba số nào lập thành cấp số cộng. Lời giải. Ta chứng minh mệnh đề tổng quát: Từ 3 n số tự nhiên đầu tiên luôn có thể chọn ra 2n số sao cho không có ba số nào lập thành một cấp số cộng. Thật vậy xét trong hệ đếm cơ số 3 tập hợp tất cả các số có ≤ n chữ số. Chọn các số mà trong biểu diễn tam phân của nó chỉ chứa chữ số 2 và chữ số 0. Khi đó có 2 n số như vậy và không có ba số nào trong chúng lập thành cấp số cộng. Bài toán 2.5 (Singapore 1995). Cho dãy số {fn} xác định bởi
  • 47. Viết thuê đề tài giá rẻ trọn gói - KB Zalo/Tele : 0973.287.149 Luanvanmaster.com – Cần Kham Thảo - Kết bạn Zalo/Tele : 0973.287.149 f1 = 1, f2n = fn, và f2n+1 = f2n + 1.
  • 48. Viết thuê đề tài giá rẻ trọn gói - KB Zalo/Tele : 0973.287.149 Luanvanmaster.com – Cần Kham Thảo - Kết bạn Zalo/Tele : 0973.287.149 23 i) Tính M = max {f1, f2, . . . , f1994} . ii) Tìm tất cả các giá trị của n, 1 ≤ n ≤ 1994 sao cho fn = M. Lời giải. Có thể dùng quy nạp để chứng minh rằng fn là số tất cả các chữ số 1 trong biểu diễn nhị phân của số n. i) Tồn tại nhiều nhất là 10 chữ số 1 trong biểu diễn nhị phân của một số nếu số đó bé hơn hoặc bằng 1994 = 11111001010(2) suy ra M = 10. ii) Với mọi số tự nhiên n ≤ 1994, ta có fn = 10 nếu và chỉ nếu n là một trong các số 1023 = 1111111111(2) 1535 = 10111111111(2) 1791 = 11011111111(2) 1919 = 11101111111(2) 1983 = 11110111111(2). Bài toán 2.6. Cho dãy số {an} xác định bởi ≤ 0 0 a < 1, an = 2an−1 nếu 2an−1 < 1 (2.5) 2an−1 − 1 nếu 2an−1 ≥ 1 Hỏi có bao nhiêu giá trị a0 sao cho a5 = a0. Lời giải. Khi tính an theo an−1 từ quan hệ (2.5) ta có thể lựa chọn một trong hai công thức. Tất nhiên, với a0 đã chọn rồi thì tất cả các bước tiếp theo đều xác định một cách duy nhất. Tuy nhiên, ta có thể chọn a0 như thế nào đó để sau đó các công thức tính theo đúng kịch bản đã cho. Có 2 5 = 32 kịch bản như vậy. Ví dụ với kịch bản (1, 1, 2, 1, 2) ta có x1 = 2x0, x2 = 2x1 = 4x0, x3 = 2x2 − x1 = 8x0 − x1, x4 = 2x3 = 16x0 − 2, x5 = 2x4 − 1 = 32x0 − 3.
  • 49. Viết thuê đề tài giá rẻ trọn gói - KB Zalo/Tele : 0973.287.149 Luanvanmaster.com – Cần Kham Thảo - Kết bạn Zalo/Tele : 0973.287.149 Giải phương trình x5 = x0 ta được x0 = 31 3 . Tất nhiên để có một lời giải hoàn chỉnh, ta cần phải lập luận chặt chẽ để thấy rằng các x0 thu được là khác nhau và với mỗi x0 thu được, dãy số sẽ đi đúng như kịch bản đã định. Tuy nhiên, phân tích này hướng
  • 50. Viết thuê đề tài giá rẻ trọn gói - KB Zalo/Tele : 0973.287.149 Luanvanmaster.com – Cần Kham Thảo - Kết bạn Zalo/Tele : 0973.287.149 24 chúng ta tới hệ nhị phân. Và ta có lời giải đẹp như sau: Giả sử a0 = (0d1d2d3 . . . )2 thì a1 = (0d2d3d4 . . . )2. Thật vậy - Nếu 2a0 < 1 thì d1 = 0 và a1 = 2a0 = (0d2d3d4 . . . )2. - Nếu 2a0 ≥ 1 thì d1 = 1 và a1 = 2a0 − 1 = (0d2d3d4 . . . )2. Hoàn toàn tương tự, ta có a2 = (0d3d4d5 . . . )2, . . . , a5 = (0d6d7d8 . . . )2. Như vậy a5 = a0 khi và chỉ khi a0 là phân số nhị phân tuần hoàn chu kỳ 5. Có 2 5 = 32 chu kỳ tuần hoàn như vậy, trong đó chu kỳ (11111)2 cho chúng ta a0 = 1 (loại). Vậy có tất cả 31 giá trị a0 thỏa mãn (2.5). Đó là 0, (00000), 0, (00001), . . . , 0, (11110). Tính sang hệ thập phân đó là các giá trị 0, 1 , 2 ,...,30 . 31 31 31 Bài toán 2.7. Xác định số hạng tổng quát của dãy {xn}, biết x1 = a và xm+n = xm + xn + mn, ∀m, n ∈ N∗ . (2.6) Lời giải. Giả sử tồn tại dãy số {xn} thỏa mãn điều kiện bài toán. Từ phương trình (2.6) suy ra xn+1 − xn = a + n (2.7) x1 = a Phương trình xn+1 − xn = a + n là một phương trình sai phân tuyến tính không thuần nhất, cấp một. Vì phương trình đặc trưng là λ = 1 nên ta có nghiệm tổng quát của phương trình thuần nhất xn+1 − xn = 0 như sau: xn = c. Nghiệm riêng của (2.7) có dạng x∗ n = n(dn + e). Thay x∗ n vào (2.7), ta được (n + 1) [d(n + 1) + e] − n (dn + e) = a + n 1 ⇔ ⇐ d = 2 2dn + d + e = a + n 1 1 1 Suy ra xn ∗ = e = a − 2 n2 + a − n. Vì xn = xn + xn ∗ nên nghiệm phương trình (2.7) là 2 2 1 1
  • 51. Viết thuê đề tài giá rẻ trọn gói - KB Zalo/Tele : 0973.287.149 Luanvanmaster.com – Cần Kham Thảo - Kết bạn Zalo/Tele : 0973.287.149 xn = c + n 2 + a − n. 2 2
  • 52. Viết thuê đề tài giá rẻ trọn gói - KB Zalo/Tele : 0973.287.149 Luanvanmaster.com – Cần Kham Thảo - Kết bạn Zalo/Tele : 0973.287.149 25 Vì x1 = a nên ta có c = 0. Suy ra 1 1 xn = n 2 + a − n. 2 2 1 n 2 1 Thử lại, ta thấy nghiệm xn = + a − n. Thỏa mãn điều kiện bài toán. 2 2 Bài toán 2.8. Tồn tại hay không một dãy số {xn}, thỏa mãn điều kiện xm+n = xm + xn + m + n, ∀m, n ∈ N ∗ . (2.8) Lời giải. Giả sử tồn tại dãy số {xn} thỏa mãn điều kiện bài toán. Từ phương trình (2.8) cho m = 1 suy ra xn+1 = xn + x1 + n + 1, hay xn+1 − xn = a + n, với a = x1 + 1. (2.9) Phương trình xn+1 − xn = a + n là một phương trình sai phân tuyến tính không thuần nhất, cấp một. Vì phương trình đặc trưng là λ = 1 nên ta có nghiệm tổng quát của phương trình thuần nhất xn+1 − xn = 0 là xn = c. Nghiệm riêng của (2.9) có dạng x∗ n = n(dn + e), (d = 0). Thay x∗ n vào (2.9) ta được (n + 1) [d(n + 1) + e] − n(dn + e) = a + n ⇔2dn + d + e = a + n 1 1 ⇔ d = 2 e = a − 2 Suy ra xn ∗ = 1 n 2 + a − 1 n. Vì xn = xn + xn ∗ nên ta có nghiệm của (2.9) là 2 2 1 n 2 + 1 (2.10) xn = c + a − n. 2 2 1 n 2 + a − 1 Vì x1 = a − 1 nên từ (2.10) ta có c = −1. Vì thế xn = n − 1. 2 2
  • 53. Viết thuê đề tài giá rẻ trọn gói - KB Zalo/Tele : 0973.287.149 Luanvanmaster.com – Cần Kham Thảo - Kết bạn Zalo/Tele : 0973.287.149 Thử lại ta thấy nghiệm này không thỏa mãn bài ra. Vậy không tồn tại dãy số thỏa mãn bài ra.
  • 54. Viết thuê đề tài giá rẻ trọn gói - KB Zalo/Tele : 0973.287.149 Luanvanmaster.com – Cần Kham Thảo - Kết bạn Zalo/Tele : 0973.287.149 26 Bài toán 2.9. Xác định dãy số dương {xn}, thỏa mãn điều kiện xmn = xmxn, ∀m, n ∈ N ∗ . (2.11) Lời giải. Giả sử tồn tại dãy số dương {xn} thỏa mãn (2.11). Ta có x1.n = x1xn ⇒ x1 = 1. Nhận xét rằng. Nếu n = pk với p là số nguyên tố thì xn = xpk = (xp) k . Ta chứng minh nhận xét trên bằng quy nạp. Với k = 1 ta có xn = x 1 p = (xp) 1 . Giả sử nhận xét đúng với k = q, (q ≥ 1). Khi đó, với n = p k+1 , ta có xn = xpk+1 = xpk xp = (xp)k xp = (xp)k+1 . Do đó với n = pk thì xn = xpk = (xp)k . Từ đây suy ra, nếu n = pm 11 . . . pm ss thì xn = (xp1 )m1 . . . (xps )m s . Vậy xp có thể nhận giá trị dương tùy ý khi p là một số nguyên tố. Do vậy, ta kết luận như sau: xp có thể nhận giá trị dương tùy ý khi p là một số nguyên tố và xn = (xp1 )m1 . . . (xps )ms . khi n = pm 11 . . . pm ss . Bài toán 2.10 (IMO 2009). Giả sử s1, s2, · · · , sn, · · · là một dãy số tăng nghiêm ngặt các số nguyên dương thỏa mãn: Các dãy con ss1 , ss2 , · · · , ssn , · · · và ss1+1, ss2+1, · · · , ssn+1, · · · là các cấp số cộng. Chứng minh rằng bản thân dãy s1, s2, · · · , sn, · · · cũng là cấp số cộng. Lời giải. Để dễ sử lí, ta phát biểu bài toán dưới dạng. Cho f : N ∗ → N ∗ là hàm tăng thực sự và thỏa mãn: Các dãy số (f (f (n))) và f (f (n) + 1) là cấp số cộng. Chứng minh rằng dãy (f (n)) cũng là cấp số cộng. Từ giả thiết suy ra f (f (n)) = an + b và f (f (n)+) = cn + d. Với a, c là những số nguyên dương, b, d là các số nguyên, ta có f (f (n)) < f (f (n) + 1) ≤ f (f (n + 1)). Suy ra an + b < cn + d ≤ a(n + 1) + b hay a + n b < c + n d ≤ a + a + n b . Bằng cách cho n → +∞ ta được a = c. Tóm lại ta có
  • 55. Viết thuê đề tài giá rẻ trọn gói - KB Zalo/Tele : 0973.287.149 Luanvanmaster.com – Cần Kham Thảo - Kết bạn Zalo/Tele : 0973.287.149 f (f (n)) = an + b, f (f (n)+) = an + c, ∀n ∈ N ∗ .
  • 56. Viết thuê đề tài giá rẻ trọn gói - KB Zalo/Tele : 0973.287.149 Luanvanmaster.com – Cần Kham Thảo - Kết bạn Zalo/Tele : 0973.287.149 27 Dễ dàng chứng minh được f (an + b) = af (n) + b và f (an + b + 1) = af (n) + c. Ta có 0 < f (n + 1) − f (n) ≤ a, ∀n ∈ N ∗ . Gọi r, t lần lượt là các số tự nhiên sao cho M = f (r + 1) − f (r) lớn nhất và m = f (t + 1) − f (t) nhỏ nhất. Ta có M = f (f (f (r)) + 1) − f (f (f (r))) = f (ar + b + 1) − f (ar + b) = c − b, m = f (f (f (t)) + 1) − f (f (f (t))) = f (at + b + 1) − f (tr + b) = c − b. Vậy M = m. Suy ra f (n + 1) − f (n) = c − b, ∀n ∈ N∗ . Từ đó ta có điều phải chứng minh. 2.3 Các bài tập áp dụng Các kết quả của các bài toán phương trình hàm số học có ý nghĩa rất lớn trong việc giải các bài toán phương trình hàm trên tập các số tự nhiên. Bằng các phép biến đổi đưa về các phương trình hàm đã xét ở trên. Sau đây ta đi xét một số bài tập áp dụng các kết quả kể trên. Bài toán 2.11. Cho hàm số f : N ∗ → N ∗ , thỏa mãn điều kiện f (mf (n)) = n2 f (m), ∀m, n ∈ N∗ . Xác định giá trị nhỏ nhất có thể có của f (2010). Lời giải. Cho m = n = 1 ta có f (f (1)) = f (1), suy ra f (1) = 1. Cho m = 1 ta có f (f (n)) = n2 , ∀n ∈ N∗ . Thay m bởi f (m) ta được f (f (m)f (n)) = n 2 f (f (m)) = n 2 m 2 = f (f (mn)), ∀m, n ∈ N ∗ . Do f đơn ánh nên f (mn) = f (m)f (n), ∀m, n ∈ N ∗ . Ta chỉ cần xác định giá trị hàm số tại các số nguyên tố. Giả sử p là số nguyên tố và f (p) = ab, (a > 1, b > 1).
  • 57. Viết thuê đề tài giá rẻ trọn gói - KB Zalo/Tele : 0973.287.149 Luanvanmaster.com – Cần Kham Thảo - Kết bạn Zalo/Tele : 0973.287.149 Khi đó f (ab) = f (f (p)) hay f (a)f (b) = p 2 .
  • 58. Viết thuê đề tài giá rẻ trọn gói - KB Zalo/Tele : 0973.287.149 Luanvanmaster.com – Cần Kham Thảo - Kết bạn Zalo/Tele : 0973.287.149 28 Do p là nguyên tố và f (a) > 1, f (b) > 1 nên chỉ xảy ra khả năng f (a) = f (b) = p, suy ra a = b. Vậy f (p) = a 2 và f (a) = p. Ta sẽ chứng minh a là một số nguyên tố. Thật vậy nếu a = mn, m, n > 1 thì p = f (a) = f (m)f (n), vô lí vì f (m) > 1 và f (n) > 1. Tóm lại, nếu p là số nguyên tố thì f (p) hoặc là số nguyên tố, hoặc là bình phương của một số nguyên tố. Chú ý rằng không thể xảy ra trường hợp f (p) hoặc f (p) = p 2 với p là số nguyên tố. Từ đó có thể xây dựng hàm số thỏa mãn yêu cầu bài toán như sau - Chia tập hợp số nguyên tố thành vô hạn các cặp (p; q) rời nhau và đặt f (p) = q, f (q) = p2 . - Với mọi số n = p α 11 p α 22 ...p α kk ta đặt f (n) = f (p1) α 1 f (p2) α 2 ...f (pk) α k . Ta có 2010 = 2 2 .3.5.67 nên giá trị nhỏ nhất có thể có của f (2010) ứng với hàm số f xác định bởi f (2) = 3, f (3) = 4, f (5) = 7, f (67) = 11 (các giá trị khác tùy ý miễn thỏa mãn cách xây dựng trên). Khi đó f (2010) = 3 2 .4.7.11 = 2772. Bài toán 2.12. Tìm tất cả các hàm số f : N → N, thỏa mãn điều kiện f (m + f (n)) = f (m) + n, ∀m, n ∈ N. Lời giải. Dễ thấy f là một đơn ánh. Cho m = n = 0 được f (f (0)) = f (0) ⇒ f (0) = 0. Cho m = 0 ta được f (f (n)) = n, ∀n ∈ N. Bây giờ, thay m bởi f (m) ta có f (f (m) + f (n)) = f (f (m)) + n = m + n = f (f (m + n)), ∀m, n ∈ N. Do f là đơn ánh nên f (m + n) = f (m) + f (n), ∀m, n ∈ N. Suy ra f có dạng f (n) = kn, ∀k, n ∈ N.
  • 59. Viết thuê đề tài giá rẻ trọn gói - KB Zalo/Tele : 0973.287.149 Luanvanmaster.com – Cần Kham Thảo - Kết bạn Zalo/Tele : 0973.287.149 Thay biểu thức của hàm số vào biểu thức ban đầu, ta tính được k = 1.
  • 60. Viết thuê đề tài giá rẻ trọn gói - KB Zalo/Tele : 0973.287.149 Luanvanmaster.com – Cần Kham Thảo - Kết bạn Zalo/Tele : 0973.287.149 29 Từ đó suy ra f (n) = n, ∀n ∈ N. Thử lại thấy hàm này thỏa mãn. Vậy f (n) = n, ∀n ∈ N là hàm số cần tìm. Bài toán 2.13 (Pan African 2010). Tồn tại hay không hàm số f : N → Z thỏa mãn điều kiện f (x + f (y)) = f (x) − y, ∀x, y ∈ N. (2.12) Lời giải. Giả sử tồn tại hàm f thỏa mãn (2.12). Ta chứng minh f là song ánh. Giả sử f (x1) = f (x2). Khi đó: do (2.12) do (2.12) f (x) − x1 = f (x + f (x1)) = f (x + f (x2) = f (x) − x2 ⇒ x1 = x2. Vậy f là đơn ánh. Giả sử t ∈ N, khi đó tồn tại k = 1 + f (f (1) − t) ∈ N và do (2.12) do (2.12) f (k) = f (1 + f (f (1) − t)) = f (1)− [f (1) − t] = t. Vậy f là toàn ánh, suy ra f là song ánh. Từ (2.12) cho x = 1 và y = 0, ta được f (1 + f (0)) = f (1). Mà f là song ánh nên từ đây suy ra: 1 + f (0) = 1 hay f (0) = 0. Từ (2.12) cho x = 0, ta được: f (f (y)) = −y, ∀y ∈ N. Như vậy (2.12) trở thành f (x + f (y)) = f (x) + f (f (y)), ∀x, y ∈ N. (2.13) Mà f là song ánh nên từ (2.13) ta suy ra f (x + y) = f (x) + f (y), ∀x, y ∈ N. (2.14) Từ (2.14), áp dụng kết quả bài toán 2.1, suy ra f (x) = ax, ∀x ∈ N, (a = f (1) là hằng số nguyên ). Thay vào (2.12) ta được: a(x + ay) = ax − y, ∀x, y ∈ N ⇔a 2 y = −y
  • 61. Viết thuê đề tài giá rẻ trọn gói - KB Zalo/Tele : 0973.287.149 Luanvanmaster.com – Cần Kham Thảo - Kết bạn Zalo/Tele : 0973.287.149 ⇔a 2 = −1. Điều này vô lí chứng tỏ không tồn tại hàm f thỏa mãn bài ra.
  • 62. Viết thuê đề tài giá rẻ trọn gói - KB Zalo/Tele : 0973.287.149 Luanvanmaster.com – Cần Kham Thảo - Kết bạn Zalo/Tele : 0973.287.149 30 Bài toán 2.14 (France Selection Test 2007). Tìm tất cả các hàm số f : N → Z thỏa mãn điều kiện f (x − y + f (y)) = f (x) + f (y), ∀x, y ∈ N. (2.15) Lời giải. Kí hiệu P (u, v) chỉ phép thay x bởi u và y bởi v vào (2.15). Đặt f (0) = a. Khi đó ta có P (0, 0) ⇒ f (a) = 2a ⇒ f (a) − a = a, P (0, a) ⇒ f (a) = a + f (a) ⇒ a = 0 ⇒ f (0) = 0, và P (0, x) ⇒ f (f (x) − x) = f (x), ∀x ∈ N. (2.16) Thực hiện P (x, f (y) − y) ta được f (x − f (y) + y + f (f (y) − y)) = f (x) + f (f (y) − y), ∀x, y ∈ N. (2.17) Thay (2.16) vào (2.17) ta được f (x + y) = f (x) + f (y), ∀x, y ∈ N. Từ đây áp dụng kết quả bài toán 2.1, ta suy ra f (x) = ax, ∀x ∈ N (a = f (1) là hằng số nguyên ). Thay vào (2.15) ta được a(x − y + ay) = ax + ay, ∀x, y ∈ N ⇔(a 2 − 2a)y = 0, ∀y ∈ N. Vậy a ∈ {0, 2}, do đó có hai hàm số thỏa mãn các yêu cầu đề bài là f (x) = 0, và f (x) = 2x, ∀x ∈ N. Bài toán 2.15 (Hong Kong Team Selection Tests 2008). Giả sử hàm số f : N → Z thỏa mãn f (1) = 1, f (2) = 20, f (−4) = −4 và
  • 63. Viết thuê đề tài giá rẻ trọn gói - KB Zalo/Tele : 0973.287.149 Luanvanmaster.com – Cần Kham Thảo - Kết bạn Zalo/Tele : 0973.287.149 f (x + y) = f (x) + f (y) + axy(x + y) + bxy + c(x + y) + 4, ∀x, y ∈ N. (2.18) trong đó a, b, c là các hằng số nguyên.
  • 64. Viết thuê đề tài giá rẻ trọn gói - KB Zalo/Tele : 0973.287.149 Luanvanmaster.com – Cần Kham Thảo - Kết bạn Zalo/Tele : 0973.287.149 31 a)Tìm hàm số f. b) Giả sử f (x) ≥ mx2 + (5m + 1)x + 4m, ∀x ∈ N. Hãy tìm giá trị nhỏ nhất của m. Lời giải. Kí hiệu P (u, v) chỉ việc thay x bởi u và thay y bởi v vào (2.18) a) Ta có P (0, 0) ⇒ f (0) = −4 P (0, y) ⇒ cy = 0, ∀y ∈ N ⇒ c = 0 P (1, −1) ⇒ −4 = 1 + f (−1) − b − 4 ⇒ f (−1) = b − 9 P (−1, −1) ⇒ f (−2) = 2f (−1) − 2a + b + 4 = −2a + 3b − 14 P (−2, −2) ⇒ f (−4) = 2f (−2) − 16a + 4b + 4 = −20a + 10b − 24 Do đó − 4 = −20a + 10b − 24 ⇔20a − 10b = −20 ⇔2a − b = −2. (2.19) Và P (1, 1) ⇒ f (2) = 2f (1) + 2a + b + 4 ⇒ 2a + b = 14. (2.20) Từ (2.19) và (2.20) suy ra a = 3, b = 8. Vậy f (x + y) = f (x) + f (y) + 3xy(x + y) + 8xy + 4, ∀x, y ∈ N. (2.21) Xét hàm số g : N → Z như sau g(x) = f (x) − (x 3 + 4x 2 − 4) ⇔f (x) = g(x) + (x 3 + 4x 2 − 4). Thay vào (2.21), suy ra, với mọi x, y ∈ N, ta có g(x + y) + x 3 + y 3 + 3xy(x + y) + 4(x + y) 2 − 4 = = g(x) + g(y) + (x 3 + 4x 2 − 4) + (y 3 + 4y 2 − 4) + 3xy(x + y) + 8xy + 4 = g(x) + g(y) + x 3 + y 3 + 3xy(x + y) + 4(x + y) 2 − 4. Như vậy g(x + y) = g(x) + g(y), ∀x, y ∈ N. Từ đây áp dụng kết quả bài toán 2.1, suy ra
  • 65. Viết thuê đề tài giá rẻ trọn gói - KB Zalo/Tele : 0973.287.149 Luanvanmaster.com – Cần Kham Thảo - Kết bạn Zalo/Tele : 0973.287.149 g(x) = ax, ∀x ∈ Z (a = g(1) là hằng số nguyên ).
  • 66. Viết thuê đề tài giá rẻ trọn gói - KB Zalo/Tele : 0973.287.149 Luanvanmaster.com – Cần Kham Thảo - Kết bạn Zalo/Tele : 0973.287.149 32 Mà g(1) = f (1) − 1 = 0 nên a = 0. Do đó g(x) = 0, ∀x ∈ N hay f (x) = x 3 + 4x 2 − 4, ∀x ∈ N. Thử lại thấy hàm số trên thỏa mãn yêu cầu bài ra. b)f (x) ≥ mx 2 + (5m + 1)x + 4m, ∀x ∈ N. Khi đó x 3 + 4x 2 − 4 ≥ mx 2 + (5m + 1)x + 4m, ∀x ∈ N ⇔x 3 + 4x 2 − x − 4 ≥ m(x 2 + 5x + 4), ∀x ∈ N ⇔(x + 1)(x − 1)(x + 4) ≥ m(x + 1)(x + 4), ∀x ∈ N ⇔x − 1 ≥ m, ∀x ∈ N Vậy giá trị nhỏ nhất của m là −1, đạt được khi x = 0.
  • 67. Viết thuê đề tài giá rẻ trọn gói - KB Zalo/Tele : 0973.287.149 Luanvanmaster.com – Cần Kham Thảo - Kết bạn Zalo/Tele : 0973.287.149 33 Chương 3 Các dạng toán liên quan